Sie sind auf Seite 1von 67

RMO MATERIALS

GEOMETRY FOR MATHS OLYMPIAD


1. Angles:

(a) Two rays OA and OB with common point O form an angle AOB denoted as AOB .
(b) Angle can be measured by degrees, minute and second. 10=60’, 1’ = 60’’.
(c) Right Angle: An angle whose measurement is 900 is called as right angle.
(d) Acute angle: An angle whose measurement is greater than 00 and less than 900 is
called as acute angle..
(e) Obtuse angle: An angle whose measurement is greater than 900 and less than 1800 is
called as obtuse angle.
(f) Straight angle: An angle whose measurement is 1800 is called as straight angle.
(g) Reflex angle: An angle whose measurement is greater than 1800 and less than 3600 is
called as reflex angle.
(h) Complete angle: An angle whose measurement is 3600 is called as complete angle.
(i) Equal Angles: Two angles are said to be equal if they have equal measurement.
(j) Complementary angles: Two angles are said to be complementary if their sum is 900.
(k) Supplementary angles: Two angles are said to be supplementary if their sum is 1800.
(l) Adjacent angles: Tow angles are said to be adjacent if they have one common arm
and one common vertex.
(m) If a ray stands on a straight line, then sum of two adjacent angles is 1800.
(n) The sum of all angles around a point is 3600.
(o) Vertically opposite angles: If two lines intersect at a point, then two pairs of
vertically opposite angles so formed are equal to each other.
(p) Parallel Lines: If two lines on the same plane do not intersect when produced either
side are called as parallel lines.
(q) Angles Formed when a transversal line intersect two lines:

(r) When a transversal line intersect two parallel lines:


Pairs of Alternate Interior angles are equal.
Pairs of Alternate Exterior angles are equal.
Pairs of corresponding angles are equal.
Pairs of interior angles on the same side of the transversal are supplementary.
Pairs of exterior angles on the same side of the transversal are supplementary.

Dr. Shyam Sundar Agrawal 1


RMO MATERIALS

2. A given straight line can be divided internally or externally in a given ratio at one end only
one point.
3. Harmonic Division: - If a straight line AB is divided internally at X and externally at Y in
AX AY
the same ratio i.e. if  , then we say that X and Y divide AB harmonically.
XB BY
4. IF a line AB is divided harmonically at X and Y and O is the midpoint of AB then:
(a) XY divide harmonically at A and B.
(b) OA2  OB 2  OX .OY and conversely.
1 1 2
(c)  
AX AY AB
5. Incentre: The point of concurrency of angle bisectors of a triangle is called as Incentre.
Incentre of the triangle always lies within the triangle.
6. Circumcentre: The point of concurrency of perpendicular bisectors sides of a triangle is
called as Circumcentre.
In acute angled triangle the circumcentre lies within the triangle.
In a right triangle the Circumcentre lies on the hypotenuse of the right triangle.
In an obtuse angled triangle circumcentre lies outside the triangle.
7. Orthocentre: The point of concurrency of altitudes of a triangle is called as Orthocentre.
In acute angled triangle the orthocentre lies within the triangle.
In a right triangle the orthocentre is the vertex opposite to the hypotenuse of the right
triangle.
In an obtuse angled triangle orthocentre lies outside the triangle.
8. Centroid: The point of concurrency of medians of a triangle is called as Centroid.
Centroid of the triangle always lies inside the triangle.
Centroid of the triangle divides the median in the ratio 2:1.
9. Congruence Triangles: Two triangles are congruent if they have same shape and size. For
congruent triangles their corresponding sides and angles are equal.
10. SSS Congruent Condition: Two triangles are congruent if three sides of one triangle are
respectively equal to the three sides of the other triangle.
11. SAS Congruent Condition: Two triangles are congruent if two sides and included angle of
one triangle are respectively equal to the two sides and included angle of the other triangle.
12. ASA Congruent Condition: Two triangles are congruent if two angles and included side of
one triangle are respectively equal to the two angles and included side of the other triangle.
13. RHS Congruent Condition: Two right triangles are congruent if the hypotenuse and one of
the side of one triangle are respectively equal to the three hypotenuse and one of the side of
the other triangle.
14. Similar Triangles: Two triangles are said to be similar if they have same shape but sizes are
different. If two triangles are similar then, their corresponding angles are equal and ratio of
their corresponding sides are equal.
15. AA Similarity Criteria: If two angles of one triangle are equal to the corresponding two
angles of another triangle, then that two triangles are similar.
16. SSS Similarity Criteria: If all the corresponding sides of two triangles are equal in ratio,
then two triangles are similar.
17. SAS Similarity Criteria: Two sides have lengths in the same ratio, and the angles included
between these sides have the same measure, then that two triangles are similar to each other.

Dr. Shyam Sundar Agrawal 2


RMO MATERIALS

18. If A, B, C , D , … and A, B, C , D , … are two sets of points on two parallel straight lines
such that: AB : AB   BC : B C   CD : C D  ... then the lines , AA, BB, CC , DD... are
concurrent.

19. Cevas’ Theorem: If three concurrent straight lines are drawn from the angular point of a
triangle to meet the opposite sides, then the product of three alternate segments taken in
order is equal to the product of the other three segments.

BD.CE . AF  DC.EA.FB

20. Menulas’ Theorem: If a transversal drawn to cut the sides, or sides produced, of a triangle,
the product of three alternate segments taken in order is equal to the product of the other
three segments.

AF .BD.CE  EA.FB.DC

21. Stewart’s Theorem: From the figure: a( p 2  mn)  b 2 m  a 2 n .

22. The distance of any vertex of a triangle from the orthocenter is double the distance of the
opposite side of the circumcentre.
23. Properties of Triangle:

(a) Sum of all the interior angles of a triangle is 180 0 .

Dr. Shyam Sundar Agrawal 3


RMO MATERIALS

(b) The longest side of a triangle is opposite to the greatest angle and vice versa. The
least side of triangle is opposite to the smaller angle and vice versa.
(c) Sum of any two sides of a triangle is always greater than the third side.
(d) If one angle of a triangle is greater than the second angle then the side opposite to the
first is greater than the side opposite to second.
(e) Exterior angle of a triangle is equal to the sum of two interior opposite angles.
(f) A scalene triangle is one which has all sides are unequal.
(g) An isosceles triangle has two of its sides are equal.
(h) An equilateral triangle has all the three sides are equal.
(i) An obtuse angled triangle has one of its angle is obtuse.
(j) An acute angled triangle has all of its angles are acute.
(k) A right angled triangle has one right angle.
(l) There can be only one right angle in a triangle.
(m) There can be only one obtuse angle in a triangle.
(n) Angles opposite to equal sides of a triangle are equal.
(o) Sides opposite to equal angles of a triangle are equal.
(p) The line segment joining the mid points of two sides of a triangle is parallel to third
and half of third side.
(q) A line drawn through the midpoint of two sides of a triangle is parallel to the third
side and half of third side.
(r) In a triangle ABC, if BC 2  AB 2  AC 2 , then A is right angle.
(s) In a triangle ABC, if BC 2  AB 2  AC 2 , then A is acute angle.
(t) In a triangle ABC, if BC 2  AB 2  AC 2 , then A is obtuse angle.
(u) Triangles on the same base and same parallels are equal in area.
(v) If two triangles, have a common vertex and their bases are along the same straight
line, then ratio of their areas is equal to the ratio of their base.
(w) In any triangle the sum of squares on two sides of a triangle is equal to the twice the
sum of the squares on half the third side and median to the third side.
(x) In a triangle ABC:
(1) If I be the incentre i.e. point of intersection of angle bisectors then:
BIC  90 0  12 BAC , AIC  90 0  12 ABC , AIB  90 0  12 ACB
(2) If O be the circumcentre i.e. point of intersection of side bisector then:
BOC  2BAC , AOC  2ABC , AOB  2ACB
(3) If H be the orthocenter i.e. point of intersection of altitudes then:
BHC  180 0  BAC , AHC  180 0  ABC , AHC  180 0  ACB

24. Properties of Parallelogram:

(a) Opposite sides of a parallelogram are equal.


(b) Opposite angles of a parallelogram are equal.
(c) A pair of adjacent angles of a parallelogram is supplementary.
(d) A diagonal divided the parallelogram into two congruent triangles.
(e) Diagonals of a parallelogram bisect each other.
(f) If a parallelogram can be inscribed in a circle, then it is a rectangle.
(g) If two parallelograms are on same base and same parallels then they are equal in area.

Dr. Shyam Sundar Agrawal 4


RMO MATERIALS

25. Properties of Rectangle:

(a) Opposite sides of a rectangle are equal.


(b) Each angles of a rectangle is right angle.
(c) Diagonals of a rectangle bisect each other.
(d) Diagonals of a rectangle are equal.

26. Properties of a square:

(a) All the angles of a square are right angles.


(b) All the sides of a square are equal.
(c) Diagonals of a square are equal.
(d) Diagonal bisect each other.
(e) Diagonals of a square are perpendicular to each other.

27. Properties of Rhombus:

(a) All the sides of the rhombus are equal.


(b) Opposite angles of the rhombus are equal.
(c) Diagonal of a rhombus bisect each other.
(d) Diagonals of a rhombus are perpendicular to each other.
(e) Diagonal bisects the angles at the vertices.
(f) If a rhombus can be inscribed in a circle, then it is a square.

28. Properties of Parallel Lines:

(a) Two lines in a plane which are parallel to the given line is parallel to each other.
(b) Two intersecting lines cannot both parallel to the same line.
(c) Two lines in the same plane, which are perpendicular to a given line in the same
plane, are parallel to each other.
(d) Three or more parallel lines makes equal intercepts on any transversal then they make
equal intercepts on any other transversal also.
(e) If a line parallel to one of the side of a triangle intersects other two sides in the two
different points then intercepts proportional to these two sides.
(f) If three or more parallel lines intersects two transversal, then the intercepts made on
one transversal are proportional to the intercepts made on other transversal.

29. Properties of a Circle;

(a) One and only one circle can pass through three non-collinear points.
(b) Perpendicular from the centre of a circle to any chord, bisect the chord.
(c) The line segment joining the centre of a circle to the midpoint of the chord is perpendicular
to the chord.
(d) Equal chords of a circle subtend equal angles at the centre.
(e) In a circle, chords which subtend equal angles at centre are equal.
(f) Equal chords of a circle are equidistant from the centre.
(g) Chords which are equidistant from centre are equal.

Dr. Shyam Sundar Agrawal 5


RMO MATERIALS

(h) An angle subtended by an arc at the centre is twice the angle subtended by it at any point on
the remaining part of the circle.
(i) If two chords subtend equal angles at the circumference of a circle, the chords are equal.
(j) Angles in the same segment of a circle are equal.
(k) Angle on a semicircle is a right angle.
(l) Angle in a segment smaller than a semi circle is obtuse.
(m) Angle in a segment greater than a semi circle is acute.
(n) The line perpendicular to the tangent at the point of contact passes through the centre of the
circle.
(o) The line drawn perpendicular to the radius through the end point where the radius meets the
circle is tangent to the circle at the end point.
(p) The lengths of the two tangents drawn from an external point to the circle are equal.
(q) If two chords of a circle intersect, whether inside or outside the circle, the rectangles
contained by the segments of chords are equal in area.
(r) If through a point on the circle, a tangents and a chord is drawn, then the angle which the
tangent makes with the chord is equal to the angle in the alternate segments.
(s) If from an external point P to a circle PAB & PCD are secant lines then
PA  PB  PC  PD .
(t) If from an external point P to a circle PAB is a secant line and PT is a tangent to the circle
then PA  PB  PT 2 .
(u) Apollonius Circle: If A and B are two fixed points and a variable point P moves such that
PA
is a constant K then the locus of the point P is a circle. This circle is called as
PB
Apollonius circle.
(v) Apollonius Theorem: If the side BC of a triangle ABC is divided at D such that
m.BD  n.DC , then mAB 2  nAC 2  (m  n) AD 2  mBD 2  nDC 2 .
(w) Nine Point Circle Theorem: A unique circle passes through the mid points of the sides of a
triangle, the feet of altitudes, the midpoints of the line joining the orthocenter to the vertices
of the triangle. The centre of the circle is at the midpoint of the straight line joining the
circumcentre and orthocenter and its radius is equal to half the circumradius. This circle is
called nine point circle of the triangle.
(x) Circum-circle of a triangle and its radius:

a b c
(1) R   .
2 sin A 2 sin B 2 sin C
abc
(2) R , where  is the area of triangle.
4
(y) In-circle of a triangle and its radius:

Dr. Shyam Sundar Agrawal 6


RMO MATERIALS


(1) r , where  is the area of triangle and s is the perimeter of triangle.
s
A B C
(2) r  4 sin sin sin .
2 2 2
A B C
(3) r  ( s  a ) tan  ( s  b) tan  ( s  c ) tan .
2 2 2
B C C A A B
a sin sin b sin sin c sin sin
(4) r 2 2  2 2  2 2 .
A B C
cos cos cos
2 2 2
r
(5) cos A  cos B  cos C  1  .
R
(z) Ex-circle of a triangle and its radius:

B C
a cos cos
 2 2  s tan A  ( s  c) cot B  ( s  c ) cot C  a
(1) r1   .
sa A 2 2 2 B C
cos tan  tan
2 2 2
C A
b cos cos
 2 2  s tan B  ( s  c) cot A  ( s  a) cot C  b
(2) r2   .
sb B 2 2 2 C A
cos tan  tan
2 2 2
Dr. Shyam Sundar Agrawal 7
RMO MATERIALS

A B
c cos cos
 2 2  s tan C  ( s  a) cot B  ( s  b) cot A  c
(3) r3   .
sc C 2 2 2 A B
cos tan  tan
2 2 2
(4) r1  r2  r3  r  4 R .
1 1 1 1
(5)    .
r1 r2 r3 r
1 1 1 1 a 2  b2  c2
(6) 2  2  2  2  .
r r1 r2 r3 2
1 1 1 1
(7)    .
bc ca ab 2 Rr
(8) r1r2  r2 r3  r3 r1  s 2 .
(9)   2 R 2 sin A sin B sin C .
A B C
(10)   4 Rr cos cos cos .
2 2 2
A B C
(11) r1  4 R sin cos cos .
2 2 2
A B C
(12) r2  4 R cos sin cos .
2 2 2
A B C
(13) r3  4 R cos cos sin .
2 2 2

(aa) In a triangle ABC, let R, r , r1 , r2 , r3 be the circumradius, inradius and exradii


opposite to A, B, C respectively and S is the circumcentre. I , I1 , I 2 , I 3 are incentre
and the excentres opposite to A, B, B respectively, then:
(1) SI 2  R 2  2 Rr
2
(2) SI1  R 2  2Rr1
2
(3) SI 2  R 2  2Rr2
2
(4) SI 3  R 2  2Rr3
(bb) Distance of Circumcentre (O) from the Orthocentre(H), Incentre (I) and
Excentre I1 , I 2 , I 3 :
(1) Distance from Circumcentre (O) and Orthocentre (H):
OH  R 1 8 cos A cos A cos C .
(2) Distance from Circumcentre (O) and Incentre (I):
A B C
OH  R 1  8 sin sin sin  R R  2r
2 2 2
(3) Distance from Circumcentre (O) and Excentre ( I1 ):
A B C
OI1  R 1  8 sin cos cos  R R  2r1
2 2 2

Dr. Shyam Sundar Agrawal 8


RMO MATERIALS

(4) Distance from Circumcentre (O) and Excentre ( I 2 ):


A B C
OI 2  R 1  8 cossin cos  R R  2r2
2 2 2
(5) Distance from Circumcentre (O) and Excentre ( I 3 ):
A B C
OI1  R 1  8 cos cos sin  R R  2r3
2 2 2

(cc) Distance of Incentre from the vertices of the Triangle:

A
Let I be the incentre. Let IP  AB . Hence IP  rPAI  .
2
A r A
From right triangle IPA, sin   AI  r cos ec
2 AI 2

A
(1) AI  r cos ec
2
B
(2) BI  r cos ec
2
C
(3) CI  r cos ec
2

(dd) Note Points:

(1) The centroid of any triangle divides the join of circumcentre and orthocentre internally in
the ratio 1:2.
(2) If H is the orthocentre of triangle ABC and AH produced meets BC at D and the
circumcentre of triangle ABC at P, then HD  DP .
As BD  c cos B and DP  BD cot C
So, DP  c cos B cot C  2 R cos B cos C .
(3) The orthocentre of an acute angled triangle is the incentre of the pedal triangle.
(4) The centre of circumcircle falls inside the triangle if triangle is acute angled but outside
when it is obtuse angled. If the triangle is right angled triangle the centre lies on midpoint
of the hypotenuse.
Dr. Shyam Sundar Agrawal 9
RMO MATERIALS

(5) The orthocentre falls inside the triangle if triangle is acute angled and outside if triangle
is obtuse angled triangle. If the triangle is right angled then orthocentre (B) lies on the
triangle.

(6) The length of median AD, BE and CF of triangle are given by:
1 1 2
(a) AD  2b 2  2c 2  a 2  b  c 2  2bc cos A .
2 2
1 1
(b) BE  2c 2  2a 2  b 2  c 2  a 2  2ca cos B .
2 2
1 1 2
(c) CF  2a 2  2b 2  c 2  a  b 2  2ab cos C .
2 2
(d) Distance between circum centre (O) and in-centre (I) of triangle ABC is
A B C
OI  R 1  8 sin sin sin .
2 2 2
(e) Distance between circum centre (O) and ex-centre ( I1 ) opposite to angle A of
A B C
triangle ABC is OI1  R 1  8 sin
cos cos .
2 2 2
(f) Distance between circum centre (O) and ex-centre ( I 2 ) opposite to angle B of
A B C
triangle ABC is OI 2  R 1  8 cos
sin cos .
2 2 2
(g) Distance between circum centre (O) and ex-centre ( I 3 ) opposite to angle C of
A B C
triangle ABC is OI 3  R 1  8 cos cos sin .
2 2 2

(ee) Cyclic Quadrilateral: If all the four vertices of a quadrilateral lies on the circumference
of the circle, then the quadrilateral is called as cyclic quadrilateral. OR A quadrilateral
ABCD is said to be cyclic quadrilateral if there exists a circle passing through all its four
vertices A, B, C and D. Let a cyclic quadrilateral with Ab  a, BC  b, CD  c, DA  d .
Let 2s  a  b  c  d .

(1) B  D  180 0 .
Dr. Shyam Sundar Agrawal 10
RMO MATERIALS

(2) A  C  180 0 .
1
(3) Area of cyclic quadrilateral = (ab  cd ) sin B .
2
(4) Area of cyclic quadrilateral = (s  a )(s  b)( s  c)(s  d ) .
a 2  b2  c2  d 2
(5) cos B 
2(ab  cd )
c  d 2  a2  b2
2
(6) cos D  .
2(ab  cd )
1 1 (ac  bd )(ad  bc )(ab  cd )
(7) Circum-radius = (ac  bd )(ad  bc)(ab  cd )  .
4 4 ( s  a)(s  b)( s  c)(s  d )
(ff) The rectangle under the diagonals of any quadrilateral is less than or equal to the sum of the
rectangles under its opposite sides:
AB.CD  AD.BC  AC.BD

(gg) Ptolemy’s Theorem: The rectangles under the diagonals of a cyclic quadrilateral is
equal to the sum of the rectangles under its opposite sides. i.e.
AC.BD  AB.CD  AD.BC

(hh) Regular Polygon: A regular polygon is a polygon which has all its sides equal and
all angles are equal.

(1) Sum of each interior angles of a regular polygon =  (n  2) .


 ( n  2)
(2) Each interior angle of a regular polygon of n sides = .
n
2
(3) Each exterior angles of a regular polygon of n sides = .
n
1 
(4) In-radius of the regular polygon = r  a cot .
2 n
Dr. Shyam Sundar Agrawal 11
RMO MATERIALS

1 
(5) Circum-radius of the regular polygon = R  a cos ec .
2 n
(6) Area of regular polygon = n  Area of triangle OAB
1 
 Area of regular polygon = na 2 cot (in terms of side ‘a’).
4 n

 Area of regular polygon = nr 2 tan (in terms of in-radius ‘r’).
n
1 2
 Area of regular polygon = nR 2 sin (in terms of circum-radius ‘R’).
2 n

1. Area Surface & Volume

Triangle:
 Perimeter of a triangle = Sum of all the three sides.
1
 Area of a triangle =  Base Height .
2
abc
 Area of a triangle = s ( s  a)( s  b)(s  c) , where s  .
2
 Perimeter of equilateral triangle = 3  side .
3 1
 Area of equilateral triangle =  ( side) 2   (height ) 2 .
4 3
3
 Height of equilateral triangle =  side .
2
2
 Side of equilateral triangle =  Height .
3
1 ( Base) 2
 Area of isosceles triangle =  Base  (equal side) 2  .
2 4
( Base) 2
 Height of isosceles triangle = (equal side) 2  .
4
 Base of isosceles triangle = 2 (equal side) 2  (height ) 2 .
( Base) 2
2
 Equal side isosceles triangle = (height )  .
4
 Hypotenuse of right angled isosceles triangle = 2  side .
Hypotenuse
 Equal Side of right angled isosceles triangle = .
2
1 1
 Area of right angled isosceles Triangle =  (equal side) 2   ( Hypotenuse) 2
2 4
Square:
 Perimeter of a square = 4  side
 Area of square = (side) 2 .
 Diagonal of a square = 2  side .
Dr. Shyam Sundar Agrawal 12
RMO MATERIALS

Rectangle:
 Perimeter of rectangle = 2  (length  breadth ) .
 Area of rectangle = length  breadth .
 Diagonal of rectangle = (length) 2  (breadth) 2 .
 Area of Four Walls = 2  height  (length  breadth) .

Parallelogram:
 Area of a parallelogram = Base  Height .

Rhombus:
1
 Area of rhombus =  Pr oduct of diagonals .
2
1
 Side of a rhombus = (d1 ) 2  (d 2 ) 2 , where d1 , d 2 are lengths of diagonals.
2

Trapezium:
1
 Area of trapezium =  Sum of parallel sides  Height
2

Circle:

 Circumference of a circle = 2 .r   .d .
 .d 2
 Area of circle =  .r 2  .
4
diameter
 Radius of circle = .
2
Arc:

 Length of arc =  2 .r .
360 0

 Area of arc =   .r 2
360 0

Cube: Let side of the cube = a .

 Surface area of cube = 6a 2 .


 Area of four walls of cube = 4a 2 .
 Volume of the cube = a 3 .
 Main diagonal of a cube = 3  a .

Cuboid: Let length = l , breadth = b and height = h

Dr. Shyam Sundar Agrawal 13


RMO MATERIALS

 Total surface area of cuboid = 2(lb  bh  hl )


 Area of four walls of cuboid = 2h(l  b) .
 Volume of the cuboid = l  b  h .
 Main diagonal of a cuboid = l 2  b 2  h2 .

Right Circular Cylinder: Let radius of base = r and height = h

 Base area of cylinder =  .r 2


 Curved surface area of cylinder = 2 .rh
 Total surface area of cylinder = 2 .rh  2 .r 2  2 .r (r  h) .
 Volume of cylinder =  .r 2 h

Right Circular Cone: Let radius of base = r , height = h and slant height = l

 Slant height = l  r 2  h 2
 Base area of cone =  .r 2
 Curved surface area of cone =  .rl
 Total Surface area of cone =  .r 2   .rl   .r (r  l ) .
1
 Volume of the cone =  .r 2 h
3
Sphere:
 Curved surface area of sphere = 4 .r 2
4
 Volume of the sphere =  .r 3 .
3

Hemisphere:

 Curved surface area of hemisphere= 2 .r 2


 Base area of hemisphere =  .r 2
 Total surface area of hemisphere = 3 .r 2
2
 Volume of the hemisphere =  .r 3 .
3

Frustum: Let h be height, l be the slant height and R and r are the radii of both the ends of a
frustum.
 Slant height of frustum = l  h 2  ( R  r ) 2
 Smaller base area of frustum =  .r 2
 Bigger base area of frustum =  .R 2
 Curved surface area of frustum =  .( R  r )l .
 Total surface are of frustum =  .r 2   .R 2   .( R  r )l   .(r 2  R 2  Rl  rl ) .
1
 Volume of frustum =  .h( R 2  r 2  Rr ) .
3
Dr. Shyam Sundar Agrawal 14
RMO MATERIALS

n-Gon: Let a n-gon is there.


 Sum of interior angles of n-gon = (n  2)  180 0 .
 Sum of Exterior angles of n-gon = 360 0
( n  2)
 For a regular n-gon each interior angles =  180 0 .
n
360 0
 For a regular n-gon each exterior angles = .
n

Counting Figures:
1. If a square is subdivided into nXn congruent squares, then total no. of suqres present =
n(n  1)(2n  1)
.
6
2. If a rectangle is subdivided into mXn congruent rectangles, and a diagonal is joined then the
diagonal crossed over m  n  hcf (m, n) no. of rectangles.
3. If a triangle is subdivided into nXn congruent triangles, then number of parallelogram present =
(n  1)n(n  1)(n  2)
.
8
4. If a triangle is subdivided into nXn congruent triangles, then number of triangles present =
n(n  2)(2n  1)
If n is even: then no. of triangles =
8
n(n  2)(2n 2  3n  1)
If n is odd: then no. of triangles = .
8
***********
**********
*********

Dr. Shyam Sundar Agrawal 15


RMO MATERIALS

GEOMETRY QUESTIONS
1. Find radius of the circle form the given figure:

2. Find area of the shaded portion:

3. Find area of the shaded portion:

4. Find sum of angles of the given star: A  B  C  D  E  ?

Dr. Shyam Sundar Agrawal 16


RMO MATERIALS

5. Express r in terms of a and b from the figure:

6. In a given triangle ABC, D and E are points on the side AB and AC respectively, such that
EBC  60 0 , EBD  20 0 , DCB  50 0 , DCE  30 0 , find BED  ?

7. Four circles of equal radius are inside A square of 4 cm side as given in the figure. Again the sides
of a smaller square are tangent to four circles. Find the area of the shaded portion.

8. From the given figure find the length of AB.

9. In the given triangle area of three portions are given 2, 3, and 4 sq units. Find the area of the shaded
portion.

Dr. Shyam Sundar Agrawal 17


RMO MATERIALS

10. From the given figure find the area of the shaded portion.

11. When a conical bottle is at rest with its flat base the level of the water in the bottle is ‘b’ unit from
the vertex and when the same conical bottle is turned upside down, then the water level is ‘a’ unit
form the base. Find the height of the conical bottle in terms of a and b.

12. From the figure find the height h in terms of a and b.

13. ABCD is a square. E is the midpoint of AB. AC and DE joined and they intersect each other at F.
Find the ratio of area of AEF and area of FDC.

Dr. Shyam Sundar Agrawal 18


RMO MATERIALS

14. In the given quarter of a circle with radius a units. A rectangle is inscribed in the quarter of the circle
such that sum of its length and breadth is b units (b > a), find the perimeter and area of the shaded
portion.

15. In a quadrilateral ABCD, BAD  90 0 , BCD  450 , ABD  40 0 , BDC  50 , AB  x units ,


then find the area of the quadrilateral ABCD.

16. Find the area of the shaded portion if the length of the side of each small square is ‘a’ unit.

17. In triangle ABC, AB = c unit, BC = a unit, AC = b unit. DE is the tangent to the circle (inscribed
circle) of the triangle ABC. Find the perimeter of the triangle CDE. (See the fig)

Dr. Shyam Sundar Agrawal 19


RMO MATERIALS

18. From the figure a rectangle is divided into four parts. Area of threeparts given. Find the area of
fourth part.

19. In the given figure ABCD is a quadrilateral. Diagonal AC and BD intersect at E. Area of traingles
AED, CED, AEB are a sq unit, b sq unit, c sq unit respectively. Find the area of the triangle BEC.

20. ABCD is a square. F, G, H, I are the mid point of side of the square. E is any point inside the square
such that EF, EG, EH and EI joined. Given that area of quadrilaterals AIEF, DIEH, CHEG are a sq
units, b sq units and c sq units respectively. Find the area of the quadrilateral BFEG.

21. Find the area of the square ABCD (from fig) where EF is perpendicular to DF and EB both such that
DF = 12 cm, EF = 3 cm, EB = 9 cm.

22. In the rectangle ABCD teo right triangles AEB and DFC are there such that AE = 7 units, EB = 24
cm, DF = 15 cm, find the length AD.

Dr. Shyam Sundar Agrawal 20


RMO MATERIALS

23. From each corner of a parallelogram, draw two line segment to connect the corner of the mid points
of the opposite sides. These eight line segments bound an octagon. Find the ratio of area of octagon
to the area of parallelogram. (see the fig)

24. The following fig contains six identical rectangles with 40 units of perimeter each. Find the
perimeter bounded by the red lines.

25. From the fig if area of red portion and blue portion are equal, then find the ratio of radius of bigger
circle to the radius of smaller circle.

26. Two squares are inscribed in the circle as shown in the fig. Find the ratio of area of bigger square to
the area of smaller square.

27. In the fig if length of side of the square is 1 unit, then find the area of the shaded portion.

Dr. Shyam Sundar Agrawal 21


RMO MATERIALS

28. A field is full of grass except the red area of rectangle with length 20 m and breadth 10 m as given in
the fig. A horse is tied at one corner of the rectangle by a rope which cannot cross over red area. If
length of the rope is L metre, then how much area the horse can graze?
(a) If L is less than equal to 10 m,
(b) If L is more than 10 m and less than equal to 20 m.
(c) If L is more than 20 m and less than equal to 30 m.
(d) If L is more than 30 m.

29. If in a triangle two angle bisectors are equal prove that triangle is isosceles. (Steiner Lehmus
Theorem)
30. From the figure find the area of the shaded portion.

31. A rectangle has sides of integer lengths (in cm) and an area 36 cm2. What is the maximum possible
perimeter of the rectangle?
32. In a triangle ABC, A  750 , B  60 0 , and CF, AD are altitudes from C and A respectively. H is the
orthocenter and O is the circumcentre. Prove that O is the incentre of the triangle CHD.
33. Find a relation among the sides of a triangle ABC if the median AM, the altitude BH and the angle
bisector CD concurrent at a point.
34. A circle of radius 5 cm passes through two adjacent vertices of a square. A rectangle to a circle is
drawn from a third vertex of the square is twice the length of the side of the square. Find the side of
the square.
b2  c2
35. If  be the area of the triangle ABC, then prove that   .
4
36. Consider an acute angled triangle ABC and let P be an interior point of triangle ABC. Suppose that
the lines BP and CP, when produced, meet AC and AB in E and F respectively. Let D be the point
where AP intersects the line segment EF and K be the foot of the perpendicular from D on BC.
Show that DK bisect EKF.

Dr. Shyam Sundar Agrawal 22


RMO MATERIALS

37. Let a, b, c be the sides of a given triangle ABC. Let  be the area of the triangle ABC. Show that:
a 2  b 2  c 2  4 3 .
38. In a triangle one of the angle is 120 0 . Rove that the triangle formed by the feet of the angle bisector
is right angled.
39. ABC is an equilateral triangle inscribed in a circle. P is any point on the minor arc BC. Prove that
PA =PB+PC.
40. A, B, C are three fixed points, B lying between A and C. Find all possible points D such that
AB.AC  AD 2 .
41. Let ABCD be a cyclic quadrilateral. Show that the incentres of the triangle ABC, BCD, CDA and
DAB form a rectangle.
42. ABCD is a quadrilateral. P, Q, R, Sare the mid points of AB, BC, CD, DA respectively. PR and SQ
intersect at L. T is any point within the quadrilateral. Prove that:
2 2 2 2 2 2 2 2 2
4 LT  LA  LB  LC  LD  TA  TB  TC  TD .
43. ABCD is a parallelogram. P is any point on AC. Through P, MN is drawn parallel to BA cutting BC
in M and AD in N and SR is drawn parallel to BC cutting BA in S and CD in R show that
[ASN]+[AMR]=[AND], where [ ] denotes the area of the rectilinear figure.
44. Inside a unit square, all isosceles triangles whose base is a side of the square, and whose vertex is
the midpoint of the opposite side are drawn. Find the area of the octagon determined by the
intersection of these four triangles.
45. In a triangle ABC, AB  AC , A  80 0 and S is the circumcentre. Bisectors of angles ACS and
ABS meet BS and CS respectively at X and Y. Find the angles of triangle AXY.
46. Let ABCD be a rectangle with AB = a, BC = b. Suppose r1 is the radius of the circle passing
through A and B and touching CD, r2 is the radius of the circle passing through B and C and
5
touching AD. Prove that r1  r2  (a  b) .
8

47. A quadrilateral ABCD is inscribed in a circle and BD bisect ABC . A and C are fixed points and B
AB  BC
moves on the circle. Prove that is a constant.
BD
48. Given the base and the vertical angle of a triangle prove that the area and the perimeter of a triangle
are maximum when the triangle is isosceles.
49. The length of the midline of a trapezoid equals 4 cm and the base angle are 400 and 500. Determine
the length of the bases if the distances of their mid points equals 1 cm.
50. ABC is a triangle. P and Q are points on AB so that 6PQ = 3AP = 2QB, R and S are points on BC so
that 6RS = 3SC = 2AR. Prove that PR and QS intersect on BC produced.
51.  
Show that for any triangle ABC, a 2  b 2  c 2  3 max a 2  b 2 , b 2  c 2 , c 2  a 2 .
52. A square sheet of paper ABCD is so folded that B falls on the midpoint M of CD. Prove that the
crease will divide BC in the ratio 5:3.
53. A square is constructed with two of its vertices on the bounding radii and two remaining vertices on
the arc of a sector of a disc of radius 10 units the sectorial angle being 600, find the area of the
square.
54. ABC is an isosceles triangle with B  C  780 , D and E are points on AB, AC respectively. Such
that BCD  24 0 and CBE  510 , find BED .

Dr. Shyam Sundar Agrawal 23


RMO MATERIALS

55. Let ABC be an acute angled triangle in which D, E, F are points in BC, CA, AB respectively such
that AD  BC , AE  EC and CF bisect C internally. Suppose CF meets AD and DE in M and N
respectively. If FM = 2, MN = 1, NC = 3, find the perimeter of the triangle ABC.
56. Three circles touch each other externally and all the three touch a line. If two of them are equal and
third has radius 4 cm. Find the radius of the equal circles.
57. Points M and N lie inside an equilateral triangle ABC such that MAB  MBA  40 0 ,
NAB  90 0 , NBA  30 0 . Prove that MN is parallel to BC.
58. Prove that the inradius of a right angled triangle with integer sides is an integer.
59. A circle is inscribed in a triangle ABC. MN is the diameter perpendicular to the base AC. Let L be
the intersection of BM with AC. Prove that AN = LC (N lies on AC).
60. Two circles have exactly two points A and B in common. Find a straight line L through A such that
the circles cut out of L chords of equal length. How many solutions can the problem have?
61. Find the maximum number of right angles which can occur among the interior angles of a convex
polygon.
62. ABCD is a square. P, Q, R, S are the mid points of AB, BC, CS and DA respectively. By joining
AR, BS, CP, DQ we get a quadrilateral. Prove that the quadrilateral is a square. What is the ratio of
the area of this square to the square ABCD.
63. How many maximum points of intersection can be got by arranging 8 straight lines and 4 circles in a
plane.
64. Find a point P inside a triangle ABC such that the product of its distances from the sides is
maximum.
65. How to inscribe a square in a given triangle so that one side may lie along a side of the triangle.
66. BCML is a quare and P is any point outside the square. Prove that PL2  PB 2  PM 2  PC 2 .
Deduce that if P is any point within the triangle ABC and squares BCML, CNRA, ASTB are
described externally on its sides, then PL2  PN 2  PS 2  PM 2  PR 2  PT 2 .
67. S is the circumcentre and O is the orthocenter of triangle ABC. Cs is produced to meet the
circumcircle at L. Prove that AO = 2ST, where ST is perpendicular to BC and LBOA is a
parallelogram.
68. PQRS is a square. T is the mid point of PQ, ST is produced to M so that ST = 5TM. Show that M
lies on the circle circumscribing the square.
69. (IMO1959) Construct a right triangle with given hypotenuse c such that the median drawn to the
hypotenuse is the geometric mean of the two legs of the triangle.
70. (IMO1959) An arbitrary point M is selected in the interior of the segment AB. The square AMCD
and MBEF are constructed on the same side of AB, with the segments AM and MB as their
respective bases. The circles circumscribed about these squares, with centers P and Q, intersect at M
and also at another point N. Let N  denote the point of intersection of the straight line AF and BC.
(a) Prove that the points N and N  coincide.
(b) Prove that the straight lines MN pass through a fixed point S independent of the choice of M.
(c) Find the locus of the midpoints of the segments PQ as M varies between A and B.
71. (IMO1959) Two planes, P and Q, intersect along the line p. The point A is given in the plane P, and
the point C in the plane Q; neither of these points lies on the straight line p. Construct an isosceles
trapezoid ABCD (with AB parallel to CD) in which a circle can be inscribed, and with vertices B
and D lying in the planes P and Q.

Dr. Shyam Sundar Agrawal 24


RMO MATERIALS

72. (IMO1960) A right angled triangle ABC is given for which the hypotenuse BC has length a and is
divided into n is odd. Let  be the angle with which the point A sees the segment containing the
4nh
middle of the hypotenuse. Prove that tan   2 , where h is the height of the triangle.
(n  1)a
73. (IMO1960) Construct a triangle ABC whose length of height ha and hb (from A and B,
respectively) and length of median ma (from A) are given.
74. (IMO1960) A cube ABCDABC D  is given.
(a) Find the locus of all the midpoints of segments XY, where X is any point on segment AC and Y
is any point on segment B D .
(b) Find the locus of all points Z on segments XY such that ZY  2 XZ .
75. (IMO1960) An isosceles trapezoid with bases a , b and height h is given.
(a) On the line of the symmetry construct the point P such that both (nonbase) sides are seen from P
with an angle of 90 0 .
(b) Find the distance of P from one of the bases of the trapezoid.
(c) Under what conditions for a , b and h can the point P be constructed (analyze all possible
cases)?
76. (IMO1960) A sphere is inscribed in a regular cone. Around the sphere a cylinder is circumscribed
so that its base is in the same plane as the base of the cone. Let V1 be the volume of the cone and V2
the volume of the cylinder.
(a) Prove that V1  V2 is impossible.
(b) Find the smallest k for which V1  kV2 , and in this case construct the angle at the vertex of the
cone.
77. (IMO1961) In the interior of P1 P2 P3 a point P is given. Let Q1 , Q2 , Q3 respectively be the
intersection of PP1 , PP2 , PP3 with the opposite edges of P1 P2 P3 . Prove that among the ratios
PP1 / PQ1 , PP2 / PQ2 , PP3 / PQ3 there exist at least one not larger that 2 and at least one not smaller
than 2.
78. (IMO1961) Construct a triangle ABC if the following elements are given: AC  b, AB  c, and
AMB  w( w  90 0 ) , where M is the mid point of BC. Prove that the construction has a solution if
w
and only if b tan  c  b . In what case does equality hold?
2
79. (IMO1961) A plane  is given and on one side of the plane three non collinear points A, B, and C
such that the plane determined by them is not parallel to  . Three arbitrary A, B, C  in  are
selected. Let L, M and N be the mid points of AA, BB, CC  and G the centroid of LMN . Find the
locus of all points obtained for G as A, B, C  are varied (independently of each other) across  .
80. (IMO1962) A cube ABCDAB C D is given. The point X is moving at a constant speed along the
square ABCD in the direction from A to B. The point Y is moving with the same constant speed
along the square BCC B  in the direction from B  to C  . Initially X and Y start out from A and B 
respectively. Find the locus of all the midpoints of XY.
81. (IMO1962) On the circle k three points A, B and C are given. Construct the fourth point on the
circle D such that one can inscribe a circle in ABCD.
82. (IMO1962) Let ABC be an isosceles triangle with circumradius r and inradius  . Prove that the
distance d between the circumcenter and incenter is given by d  r (r  2  ) .

Dr. Shyam Sundar Agrawal 25


RMO MATERIALS

83. (IMO1962) Prove that a tetrahedron SABC has five different spheres that touch all six lines
determined by its edges if and only if it is regular.
84. (IMO1963) Find the locus of points in space that are vertices of right angles of which one ray
passes through a given point and the other intersects a given segment.
85. (IMO1963) Prove that if all the angles of a convex n-gon are equal and the lengths of consecutive
edges a1 , a 2 ,...., a n satisfy a1  a2  ....  an , then a1  a2  ....  an .
86. (IMO1964) The incircle is inscribed in a triangle ABC with sides a, b and c. Three tangents to the
incircle are drawn, each of which is parallel to one side of the triangle ABC. These tangents form
three similar triangles (internal to triangle ABC) with the sides of triangle ABC. In each of these
triangles an incircle is inscribed. Determine the sum of areas of all four incircles.
87. (IMO1964) Five points are given in a plane. Among the lines that connect these five points, no two
coincide and no two are parallel or perpendicular. Through each point we construct an altitude to
each of the other lines. What is the maximal number of intersection points of these altitudes
(excluding the initial five points)?
88. (IMO1964) Given a tetrahedron ABCD, let D1 be the centroid of the triangle ABC and let A1 , B1 , C1
be the intersection of the lines parallel to DD1 and passing through the points A, B, C with the
opposite faces of the tetrahedron. Prove that the volume of the tetrahedron ABCD is one third of the
volume of the tetrahedron A1 B1C1 D1 . Does the result remain true if the point D1 is replaced with any
point inside of the triangle ABC?
89. (IMO1965) A tetrahedron ABCD is given. The lengths of the edges AB and CD are a and b
respectively, the distance between the lines AB and CD is d, and the angle between them is equal
to w . The tetrahedron divided into two parts by the plane  parallel to the lines AB and CD.
Calculate the ratios of the volumes of the parts if the ratio between the distances of the plane  from
AB and CD is equal to k.
90. (IMO1965) Given a triangle OAB such that AOB    90 0 , let M be any arbitrary point of the
triangle different from O. Denote by P and Q the feet of the perpendiculars from M to OA and OB
respectively. Let H be the orthocenter of the triangle OPQ. Find locus of the point H when:
(a) M belongs to the segment AB.
(b) M belongs to the interior of triangle OAB.
91. (IMO1966) If a, b and c are the sides and  ,  ,  the respective angles of the triangles for which

a  b  tan (a tan   b tan  ) , prove that the triangle is isosceles.
2
92. (IMO1966) Prove that the sum of distances from the center of the circumsphere of the regular
tetrahedron to its four vertices is less than the sum of the distances from any other point to the four
vertices.
93. (IMO1966) Let M, K and L be points on (AB), (BC) and (CA), respectively. Prove that the area of
at least one of the three triangles MAL, KBM and LCK is less than or equal to one-fourth the area of
triangle ABC.
94. (IMO1967) ABCD is a parallelogram; AB  a, AD  1,  is the size of the DAB , and the three
angles of the triangle ABD are acute. Prove that the four circles K A , K B , K C , K D each of radius 1,
whose centers are the vertices A, B, C and D, cover the parallelogram if and only if
a  cos  3 sin  .
95. (IMO1967) The triangle A0 B0C0 and AB C  have all their angles acute. Describe how to construct
one of the triangle ABC similar to AB C  and circumscribing A0 B0C0 (so that A, B, C corresponds

Dr. Shyam Sundar Agrawal 26


RMO MATERIALS

to corresponds to A, B , C  and AB passes through C 0 , BC passes through A0 and CA passes through
B0 ). Among these triangles ABC describe, and prove how to construct the triangle with maximum
area.
96. (IMO1968) Prove that there exists a unique triangle whose side lengths are consecutive natural
numbers and one of whose angles is twice the measure of one of the other.
97. (IMO1968) Prove that in any tetrahedron there is a vertex such that the lengths of its sides through
that vertex are sides of a triangle.
98. (IMO1969) Let AB be a diameter of a circle  . A point C different from A and B is on the circle
 . Let D be the projection of the point C onto the line AB. Consider three other circles  1 ,  2 and
 3 with the common tangent AB:  1 inscribed in the triangle ABC, and  2 and  3 tangent to both
(the segment) CD and  . Prove that  1 ,  2 and  3 have two common tangents.
99. (IMO1970) Given a point M on the side AB of the triangle ABC, let r1 and r2 be the radii of the
inscribed circles of the triangles ACM and BCM respectively while 1 and  2 be the radii of the
excircles of the triangles ACM and BCM at the sides AM and BM respectively. Let r and  denote
the respective radii of the inscribed circle and excircle at the sides AB of the triangle ABC. Prove
r r r
that: 1 2  .
1  2 
100. (IMO1970) In the tetrahedron ABCD, the edges BD and CD are mutually perpendiculars, and the
projection of the vertex D to the plane ABC is the intersection of the altitudes of the triangle ABC.
Prove that ( AB  BC  CA) 2  6( DA2  DB 2  DC 2 ) . For which tetrahedral does equality hold?
101. (IMO1971) Given a tetrahedron ABCD all of whose faces are acute angled triangles, let
  DAB  BCD  ABC  CDA . Consider all closed lines XYZTX whose vertices X, Y, Z, T
lie in the interior of segments AB, BC, CD, DA respectively. Prove that:
(a) If   0 , then there is no broken line XYZT of minimal length.
(b) If   0 , then there are infinitely many such broken lines of minimal length. That length
equals 2 AC sin( / 2) , where   BAC  CAD  DAB .
102. (IMO1973) Does there exist a finite set M of points in space, not all in the same plane, such that for
each two points A, B  M there exists two other points C , D  M such that line AB and CD are
parallel but not equal?
103. (IMO1974) Let ABC be a triangle. Prove that there exists a point D on the side AB such that CD
is the geometric mean of AD and BD if and only if sin A sin B  sin C2 .
104. (IMO1975) On the sides of an arbitrary triangle ABC, triangles BPC, CQA and ARB are externally
erected such that: PBC  CAQ  450 , BPC  QCA  30 0 , ABR  BAR  15 0 , Prove that
QRP  90 0 and QR  RP .
105. (IMO1976) A rectangular box can be filled completely with unit cubes. If one places the maximal
number of cubes with volume 2 in the box such that their edges are parallel to the edges of the box,
one can fill exactly 40% of the box. Determine all possible (interior) sizes of the box.
106. (IMO1977) Equilateral triangles ABK, BCL, CDM, DAN are constructed inside the square ABCD.
Prove that the midpoints of the four segments KL, LM, MN, NK and the midpoints of the eight
segments AK, BK, BL, CL, DM, DN, AN are twelve vertices of a regular dodecagon.
107. (IMO1978) P is a given point inside a given sphere. Three mutually perpendicular rays from P
intersect the sphere at points U, V and W; Q denotes the vertex diagonally opposite to P in the
parallelepiped determined by PU, PV and PW. Find the locus of Q for all such triads of rays from P.
Dr. Shyam Sundar Agrawal 27
RMO MATERIALS

108. (IMO1978) In triangle ABC, AB = AC. A circle is tangent internally to the circum circle of triangle
ABC and also to sides AB, AC at P, Q respectively. Prove that the midpoint of segment PQ is the
center of the incircle of triangle ABC.
109. (IMO1979) Two circles in a plane intersect. Let A be one of the points of intersection. Starting
simultaneously from A two points move with constant speeds, each point travelling along its own
circle in the same sense. The two points return to A simultaneously after one revolution. Prove that
there is a fixed point P in the plane such that, at any time, the distances from P to the moving points
are equal.
110. (IMO1979) Given a plane  , a point P in this plane and a point Q not in  , find all points R in 
such that the ratio (QP+PA)/QR is maximum.
111. (IMO1981) P is a point inside a given triangle ABC. D, E and F are the feet of the perpendicular
BC CA AB
from P to the lines BC, CA and AB respectively. Find all P for which   is least.
PD PE PF
112. (IMO1981) Three congruent circles have a common point O and lie inside a given triangle. Each
circle touches a pair of sides of the triangle. Prove that the incenter and the circumcenter of the
triangle and the point O are collinear.
113. (IMO1982) A non-isosceles triangle A1 A2 A3 is given with sides a1 , a 2 , a3 (ai is the side opposite to
Ai ) . For all i  1,2,3 , M i is the mid point of side ai , and Ti is the point where the incircle touches
side ai . Denote by S i the reflection of Ti in the interior bisector of angle Ai . Prove that the lines
M 1 S1 , M 2 S 2 and M 3 S 3 are concurrent.
114. (IMO1982) The diagonals AC and CE of the regular hexagon ABCDEF are divided by the inner
AM CN
points M and N, respectively, so that   r . Determine r if B, M and N are collinear.
AC CE
115. (IMO1983) Let K be one of the two intersection points of the circles W1 and W2 . Let O1 and O 2 be
centres of W1 and W2 . The two common tangents to the circles meet W1 and W2 respectively P1 and
P2 , the first tangent, and Q1 and Q 2 the second tangent. Let W1 and W2 be the points of P1Q1 and
P2 Q 2 respectively. Prove that O1 KO 2  M 1 KM 2 .
116. (IMO1983) Let ABC be an equilateral triangle. Let E be the set of all points from segments AB, BC
and CA (including A, B and C). Is it true that for any partition of the set E into two disjoint subsets,
there exists a right angled triangle all of whose vertices belong to the same subset in the partition?
117. (IMO1984) In a plane two different points O and A are given. For each point X  0 of the plane
denote by  ( X ) the angle AOX measured in radians (0   ( X )  2 ) and by C ( X ) the circle
(X )
with centre O and radius OX  . Suppose each point of the plane is colored by one of the
OX
finite number of colors. Show that there exists a point X with  ( X )  0 such that its color appears
somewhere on the circle C ( X ) .
118. (IMO1984) Let ABCD be a convex quadrilateral for which the circle of diameter AB is tangent to
the line CD. Show that the circle of diameter CD is tangent to the line AB if and only if the lines BC
and AD are parallel.
119. (IMO1985) A circle whose centre is on the side ED of the cyclic quadrilateral BCDE touches the
other three sides. Prove that EB  CD  ED .
120. (IMO1985) A circle with centre O passes through points A and C and intersects the sides AB and
BC of the triangle ABC at point K and N respectively. The circumscribed circles of the triangles
ABC and KBN intersect at two points B and M . Prove that OMB  90 0 .
Dr. Shyam Sundar Agrawal 28
RMO MATERIALS

121. (INMO1986) Two circles with radii a and b respectively touch each other externally. Let c be the
radius of a circle that touches these two circles as well as a common tangent to the two circles. Prove
1 1 1
that   .
c a b
122. (INMO1986) Construct a quadrilateral which is not a parallelogram, in which a pair of opposite
angles and a pair of opposite sides are equal.
123. (IMO1986) Let A, B, C be fixed points in the plane. A man starts from a certain point P0 and walks
directly to A . At A he turns his direction 60 0 to the left and walks to P1 such that P0 A  AP1 . After
he performs the same action 1986 times successively around the points A, B, C , A, B, C ,... he turns to
the starting point. Prove that ABC is an equilateral triangle, and that the vertices A, B, C arranged
counterclockwise.
124. (IMO1986) Let A, B be adjacent vertices of a regular n-gon in the plane and let O be its center.
Now let the triangle ABO glide around the polygon in such a way that the points A and B move
along the whole circumference of a polygon. Describe the figure traced by the vertex O.
125. (INMO1987) Construct the ΔABC, given ha , hb (the altitudes from A and B) and ma , the median from
the vertex A.
126. (INMO1987) Three congruent circles have a common point O and lie inside a given triangle. Each
circle touches a pair of sides of the triangle. Prove that the in-centre and the circum-centre of the
triangle and the common point O are collinear.
127. (INMO1987) Prove that any triangle having two equal internal angle bisectors (each measured from
a vertex to the opposite side) is isosceles.
128. (IMO1987) The prolongation of the bisector AL ( L  BC ) in the acute angled triangle ABC
intersects the circumscribed circle at a point N . From point L to the sides AB and AC are drawn
the perpendiculars LK and LM respectively. Prove that area of the triangle ABC is equal to the
area of quadrilateral AKNM .
129. (INMO1988) Given an QBP and a point L outside the OBP . Draw a straight line through L
meeting BQ in A and BP in C such that the triangle ABC has a given perimeter.
130. (INMO1988) Show that for a triangle with radii of circum-circle and in-circle equals to R, r
respectively, the inequality R  2r holds.
131. (IMO1988) Consider two concentric circles of radii R, r ( R  r ) with center O. Fix P on the small
circle and consider the variable chord PA of the small circle. Point B and C lie on the large circle; B,
P, C are collinear and BC is perpendicular to AP.
(a) For which value(s) of OPA is the sum of BC 2  CA 2  AB 2 extremal?
(b) What are possible positions of the midpoints U of BA and V of AC as OPA varies?
132. (IMO1988) In a right angled triangle ABC let AD be the altitude drawn to the hypotenuse and let
the straight line joining the incentres of the triangle ABD, ACD intersect the sides AB, AC at the
points K, L respectively. If E and E1 denote the areas of the triangles ABC and AKL respectively,
E
show that  2.
E1
133. (INMO1989) Triangle ABC has interior I and the incircle touches BC, CA at D, E respectively. Let
BI meet DE at G. Show that AG is perpendicular to BC.
134. (INMO1989) Let A be one of the two points of intersection of two circles with centre X and Y
respectively. The tangents at A to the two circles meet the circles again at B, C. Let a point P be
located so the PXAY is a parallelogram. Show that P is also circumcentre of triangle ABC.

Dr. Shyam Sundar Agrawal 29


RMO MATERIALS

135. (IMO1989) Let ABC be a triangle. The bisector of angle A meets the circumcircle of triangle ABC
in A1 . Points B1 and C1 are defining similarly. Let AA1 meet the lines that bisect the two external
angles at B and C in point A0 . Define B 0 and C 0 similarly. If S x1 x2 ... xn denotes the area of the
polygon x1 x 2 ...x n , prove that S A0 B 0C 0  2 S AC1BA1CB1  4 S ABC .
136. (IMO1989) The quadrilateral ABCD has the following properties:
(a) AB  AD  BC
(b) There is a point P inside it at a distance x from the side CD such that AP  x  AD and
BP  x  BC .
1 1 1
Show that   .
x AD BC
137. (RMO1990) P is any point inside a triangle ABC. The perimeter of the triangle AB + BC + CA = 2s.
Prove that s  AP  BP  CP  2 s .
138. (RMO1990) If the circum centre and centroid of a triangle coincide, prove that the triangle must be
equilateral.
139. (RMO1990) A square sheet of paper ABCD is so folded that B falls on the mid point M of CD.
Prove that the crease will divide BC in the ratio 5:3.
140. (INMO1990) Triangle ABC is scalene with angle A having a measure greater than 90 degrees.
Determine the set of points D that lie on the extended line BC, for which AD  BD . CD where
BD refers to the (positive) distance between B and D.
141. (INMO1990) Let ABC be an arbitrary acute angled triangle. For any point P lying within the
triangle, let D, E, F denote the feet of the perpendiculars from P to sides AB, BC and CA
respectively. Determine the set of all possible positions of the point P for which the triangle DEF is
isosceles. For which position of P will the triangle DEF become equilateral?
142. (IMO1990) Given a circle with two chords AB, CD that meet at E, let M be a point of chord AB
other than E. Draw the circle through D, E and M. The tangent line to the circle DEM at E meets the
AM GE
lines BC, AC at F, G respectively. Given that   , find .
AB EF
143. (RMO1991) Let P be an interior point of a triangle ABC and AP, BP, CP meets the sides BC, CA
AP AF AE
and AB in point D, E and F respectively. Show that   .
PD FB EC
144. (RMO1991) Take any point P1 on side of BC of a triangle and draw following chain of lines; P1 P2
parallel to AC, P2 P3 parallel to BC; P3 P4 parallel to AB; P4 P5 parallel to CA; P5 P6 parallel to BC.
Here P2 , P5 lie on AB; P3 , P6 lie on CA and P4 lie on BC. Show that P6 P1 is parallel to AB.
145. (INMO1991) Given any acute angled triangle ABC, let points A, B, C  , be located as follows: A is
the point where altitude from A to C on BC meets the outwards facing semicircle drawn on BC as
diameter. Points B, C  are located similarly. Prove that BCA  CAB    ABC    ABC  , where
2 2 2 2

ABC  denotes the area of triangle ABC.


146. (INMO1991) Triangle ABC has interior I. Let points X, Y be located on the line segment AB, AC
respectively so that: BX  AB  IB 2 and CY  AC  IC 2 . Given that the points X, I, Y lie on a straight
line, find the possible values of the measure of angle A.
147. (INMO1991) Triangle ABC has incentre I, its incircle touches sthe side BC at T. The line through T
parallel to IA meets the incircle again at S and the tangent to the incircle at S meets the sides AB,
AC at C , B respectively. Prove that the triangle AB C  is similar to triangle ABC.
Dr. Shyam Sundar Agrawal 30
RMO MATERIALS

1 IA.IB.IC 8
148. (IMO1991) Prove for each triangle ABC the inequality   where I is the incenter
4 l A l B lC 27
and l A , l B , lC are the lengths of the angle bisector of ABC.
149. (IMO1991) Let ABC be a triangle and M an interior point of ABC. Show that at least one of the
angle MAB, MBC , MCA is less than or equal to 30 0 .
150. (RMO1992) ABCD is a cyclic quadrilateral with AC  BD ; AC meets BD at E. Prove that
EA2  EB 2  EC 2  ED 2  4R 2 , where R is the radius of circumscribing circle.
151. (RMO1992) ABCD is a cyclic quadrilateral; x, y, z are the distances of A from the lines BD, BC and
BD BC CD
CD respectively. Prove that   .
x y z
152. (RMO1992) ABCD is a quadrilateral and P, Q are midpoints of CD and AB. AP, DQ meets at X,
and BP, CQ meets at Y. Prove that area ADX  area BCY  area quadrilate ral PXQY .
153. (RMO1992) The cyclic octagon ABCDEFGH has sides a, a, a, a, b, b, b, b respectively. Find the
radius of the circle that circumscribe ABCDEFGH.
154. (INMO1992) In a triangle ABC, angle A is twice the angle B. Show that a 2  b(b  c) .
155. (INMO1992) Two circles C1 and C 2 intersect at two distinct points P and Q in a plane. Let a line
passing through P meet the circle C1 and C 2 in A and B respectively. Let Y be the midpoint of AB
and QY meets the circle C1 and C 2 in X and Z respectively. Show that Y is also the midpoint of
XZ.
156. (IMO1992) In a plane, let there be given circle C, a line l tangent to C, and a point M on l . Find
the locus of points P that has the following property: There exist two points Q and R on l such tha
M is the midpoint of QR and C is the incenter of PQR.
157. (RMO1993) Let ABC be an acute angled triangle and CD be the altitude through c. If AB = 8, CD =
6, find the distance between the mid points of AD and BC.
158. (RMO1993) Let ABCD be a rectangle with AB = a, BC = b. Suppose r1 is the radius of the circle
passing through A and B and touching CD; and similarly r2 is the radius of circle passing through B
5
and C and touching AD. Show that r1  r2  (a  b) .
8
159. (INMO1993) The diagonals AC and BD of a cyclic quadrilateral ABCD intersect at P. Let O be the
circumcentre of triangle APB and H be the orthocenter of triangle CPD. Show that the points H, P,
O are collinear.
160. (INMO1993) Let ABC be a triangle in a plane  . Find the set of all points P(distinct from A, B, C)
in the plane  such that the circum circles of triangles ABP, BCP and CAP have the same radii.
161. (INMO1993) Let ABC be a triangle right angled at A and S be the circum circle. Let S1 be the circle
touching the lines AB and AC and the circle S internally. Further let S 2 be the circle touching the
lines AB and AC, and the circle S externally. If r1 and r2 be the radius of the circles S1 and S 2
respectively, show that r1  r2  4(areaABC ) .
162. (IMO1993) A, B, C , D are four points in the plane, with C, D on the same side of the line AB,
AB  CD
such that AC  BD  AD  BC and ADB  90 0  ACB . Find the ratio and prove that
AC  BD
circles ACD, BCD are orthogonal. (Intersecting circles are said to be orthogonal if at either common
point their tangents are perpendicular.

Dr. Shyam Sundar Agrawal 31


RMO MATERIALS

163. (IMO1993) For the three points A, B, C in a plane we define m( ABC ) to be the smallest length of
the three altitudes of the triangle ABC , where in the case of A, B, C collinear, m( ABC )  0 .
Let A, B, C be given points in the plane. Prove that for any point X in the plane,
m( ABC )  m( ABX )  m( AXC )  m( XBC ) .
164. (RMO1994) In a triangle ABC, the incicle touches the sides BC, CA and AB respectively at D, E
and F. If the radius of the circle is 4 units and if BD, CE and AF are consecutive integers, find the
sides of the triangle ABC.
165. (RMO1994) Let AC and BD be two chords of a circle with centre O such that they intersect at right
angles inside the circle at the point M. Suppose K and L are mid points of the chord AB and CD
respectively. Prove that OKML is a parallelogram.
166. (INMO 1994) Let G be the centroid of a triangle ABC in which the angle C is obtuse and AD and
CF be the median from A and C respectively onto the sides BC and AB. If the four points B, D, G
AC
and F are concylic, show that  2 . If further P is a point on the line BG extended such that
BC
AGCP is a parallelogram, show that the triangle ABC and GAP are similar.
167. (INMO 1994) A circle passes through a vertex C of a rectangle ANCD and touches its sides AB and
AD at M and N respectively. If the distance from C to the line segment MN is equal to 5 units. Find
the area of the rectangle ABCD.
168. (IMO1994) ABC is an isosceles triangle with AB = AC. Suppose that:
(a) M is the midpoint of BC and O is the point on the line AM such that OB is perpendicular to AB.
(b) Q is an arbitrary point on the segment BC different from B and C.
(c) E lies on the line AB and F lies on the line AC such that E, Q, F are distinct and collinear.
Prove that OQ is perpendicular to EF if and only if QE = QF.
169. (RMO1995) In triangle ABC, K and L are points on the sides BC (K being closer to B than L) such
that BC  KL  BK  CL and AL bisect KAC . Show that Al is perpendicular to AB.
170. (INMO1995) In an acute angled triangle ABC, mA  30 0 , H is the orthocenter and M is the
midpoint of BC. On the line HM, take a point T such that HM = MT. Show that AT = 2BC.
171. (INMO1995) Let ABC be a triangle and a circle  be drawn inside the triangle, touching its incircle
 externally and also touching the two sides AB and AC. Show that the ratio of the radii of the
  A
circles  and  is equal to tan 2  .
 4 
172. (IMO1995) Let A, B, C , D be four points on a line, in that order. The circles with diameters AC and
BD intersects at X and Y . The line XY meets BC at Z . Let P be a point on the line XY other
than Z . The line CP intersects the circle with diameter AC at C and M , and the line BP
intersects the circle with diameter BD at B and N . Prove that the lines AM , DN , XY are
concurrent.
173. (IMO1995) Let ABCDEF be a convex hexagon with AB  BC  CD and DE  EF  FA , such that
BCD  EFA   / 3 , suppose G and H are points in the interior of the hexagon such that
AGB  DHE  2 / 3 . Prove that AG  GB  GH  DH  HE  CF .
174. (RMO1996) The sides of a triangle are three consecutive integers and its inradius is four units.
Determine the circum radius.
175. (RMO1996) Let ABC be a triangle and ha be the altitude through A. Prove that
2
(b  c) 2  a 2  4ha , where a, b, c are the sides of the triangle.

Dr. Shyam Sundar Agrawal 32


RMO MATERIALS

176. (INMO1996) Let C1 and C 2 be two concentric circles in the plane with radii R and 3R respectively.
Show that the orthocenter of any triangle inscribed in circle C1 lies in the interior of circle C 2 .
Conversely, show that also every point in the interior of C 2 is the orthocenter of some triangle
inscribed in C1 .
177. (IMO1996) Let P be a point inside triangle ABC such that APB  ACB  APC  ABC . Let
D, E be the incenters of triangles APB, APC respectively. Show that AP, BD, CE meet at a point.
178. (IMO1996) Let ABCDEF be a convex hexagon such that AB is parallel to DE , BC is parallel to
EF and CD is parallel to FA . Let R A , RC , R E denote the circumradii of triangles FAB, BCD, DEF ,
P
respectively, and let P denote the perimeter of the hexagon. Prove that: R A  RC  R E  .
2
179. (JMO1997) In the given figure, AOB is a straight line. If mAOC  mCOD  100 0 , and
mBOD  mCOD  146 0 , then find out mCOD .

180. (JMO1997) In the given figure RT||SQ. If


0 0 0 0
mQPS  100 , mPQS  40 , mPSR  85 , mQRS  70 , then find out mQRT .

181. (JMO1997) In the given figure PQ||TM||RS. If mPQR  70 0 , mRTM  140 0 , then find out
mQRT .

182. (RMO1997) Let P be an interior point of a triangle ABC and let BP and CP meet AC and AB in E
and F respectively. If [BPF] = 4, [BPC] = 8, and [CPE] = 13, find [AFPE], where [] denotes area.
183. (RMO1997) In a quadrilateral ABCD, it is given that AB is parallel to CD and the diagonals AC and
BD are perpendicular to each other. Show that:
(a) AD  BC  AB  CD .
(b) AD  BC  AB  CD .
184. (INMO1997) Let ABCD be a parallelogram. Suppose a line passing through C and lying outside the
parallelogram meets AB and AD produced at E and F respectively. Show that
AC 2  CE.CF  AB. AE  AD.AF .

Dr. Shyam Sundar Agrawal 33


RMO MATERIALS

185. (IMO1997) The angle at A is the smallest angle of triangle ABC. The points B and C divide the
circum circle of the triangle into two arcs. Let U be an interior point of the arc between B and C
which does not contain A. The perpendicular bisector of AB and AC meet the line AU at V and W,
respectively. The lines BV and CW meet at T. Show that AU = TB + TC.
186. (JMO1998) Determine the perimeter of the given figure if AB = 18 cm, BC = 8cm, and angles at the
vertices, A, B, C, D, E & F are right angles.

187. (JMO1998) Find the sum of measures of the interior angles of the hexagon shown below.

188. (JMO1998) Find the number of triangles contained in the figure given below.

189. (JMO1998) If in the figure below, ABCD is a square of length 12 cm and the height of each triangle
is 4 cm (as shown in the figure), then find the area of the star.

190. (JMO1998) In the figure below, AB = AC, CD = CE, AD = DF and mBAC  40 0 . Find mAFE .

191. (JMO1998) The diagonals of a quadrilateral intersect at right angles and are of lengths 7.6 cm and
5.6 cm. Determine the areas of the quadrilateral.
192. (JMO1998) In the figure below, mB of triangle ABC is right angle. D, E, F are points on AB, Ac
and BC respectively such that AD = AE, CE = EF, determine mDEF .

Dr. Shyam Sundar Agrawal 34


RMO MATERIALS

193. (JMO1998) In the figure below, ABCD is a parallelogram. O is an interior point of parallelogram
1 1
such that mBOC  mABC and mOCB  mDCB . Determine mBOC .
3 3

194. (RMO1998) Let ABCD be a convex quadrilateral in which mBAC  500 , mCAD  60 0 ,
mCBD  30 0 and mBDC  250 . If E is the point of intersection of AC and BD, find mAEB .
195. (RMO1998) Let ABC be a triangle with AB = AC and mBAC  300 . Let A be the reflection of A
in the line BC; B  be the reflection of B in the line A; C  be the reflection of C in the line AB. Show
that A , B  , C  form the vertices of an equilateral triangle.
196. (INMO1998) In a circle C1 with centre O. Let Ab be a chord that is not a diameter. Let M be the
midpoint of AB. Take a point T on the circle C 2 with OM as diameter. Let the tangent to C 2 at T
meet C1 in P. Show that PA 2  PB 2  4PT 2 .
197. (INMO1998) Suppose ABCD is a cyclic quadrilateral inscribed in a circle of radius one unit. If
AB.BC.CD.DA  4 , prove that ABCD is a square.
198. (IMO1998) In the convex quadrilateral ABCD, the diagonals AC and BD are perpendicular and the
opposite sides AB and DC are not parallel. Suppose that the point P, where the perpendicular
bisectors of AB and DC meet, is inside ABCD. Prove that ABCD is a cyclic quadrilateral if and only
if the triangles ABP and CDP have equal areas.
199. (IMO1998) Let I be the incentre of triangle ABC. Let the incircle of ABC touch the sides BC, CA
and AB at K, L and M respectively. The line through B parallel to MK meets the lines LM and LK
at R and S respectively. Prove that angle RIS is acute.
200. (JMO1999) In the given diagram ma  350 . Find the value of mb  mc .

201. (JMO1999) In the given diagram L1 || L2 . Find the value of x, y and z.

Dr. Shyam Sundar Agrawal 35


RMO MATERIALS

202. (JMO1999) In the given diagram mA  60 0 , find the value of mOCB .

203. (JMO1999) From the given figure find the values of angle x.

204. (JMO1999) The areas of a square in cm 2 is same as its perimeter in cm. Find the length of the sides
of the square in cm.
205. (JMO1999) In the given figure lengths of sides are marked. What is the area of the figure?

206. (JMO1999) If XY  DZ in the square ABCD, then prove that XY = DZ.

207. (RMO1999)Let ABCD be a square and M, N points on sides AB, BC respectively, such that
mMDN  450 . If R is the mid point of MN show that RP = PQ where P, Q are points of
intersection of AC with lines MD, ND.
208. (RMO1999) Prove that the inradius of a right-angled with integer sides is an integer.
209. (INMO1999) Let ABC be an acute angled triangle in which D, E, F are points on BC, CA, AB
respectively such that AD is perpendicular to BC: AE = EC, and CF bisects C internally. Suppose

Dr. Shyam Sundar Agrawal 36


RMO MATERIALS

CF meets AD and DE in M and N respectively. If FM = 2, MN = 1, NC = 3, find the perimeter of


the triangle ABC.
210. (INMO1999) Let  and  be two concentric circles. Let ABC and AB C  be any two equilateral
triangles inscribed in  and  respectively. If P and P  are any two points on  and 
respectively. Show that P A 2  P B 2  P C 2  Ap 2  B P 2  C P 2 .
211. (IMO1999) Two circles G1 and G 2 are contained inside the circle G, and are tangent to G at the
distinct points M and N respectively. G1 passes through the centre of G 2 . The line passing through
the two points of intersection of G1 and G 2 meets G at A and B. The lines MA and MB meet G1 at
C and D respectively. Prove that CD is tangent to G 2 .
212. (JMO2000) The three smaller rectangles in the figure are congruent (i.e. of the same shape and size).
The length BC = 1 cm. Find out the length of AB.

213. (JMO2000) In the figure, ABCD is a square and ABE is an equilateral triangle. Find mAED  ?

214. (JMO2000) The quadrilateral PQRS is inscribed in the triangle ABC. Which has a larger perimeter:
triangle ABC or quadrilateral PQRS?

215. (JMO2000) In the triangle ABC, AB = AC and E, F are point on AB and AC respectively. EF || BC,
show that EC = BF.

216. (JMO2000) In the figure, the three equally large circles touch each other and they touch the edge of
a triangle. If the radius of each circle is 3 cm, then find the perimeter of the triangle.

217. (JMO2000) In the trapezium ABCD, AC = BD. Show that AD = BC.

Dr. Shyam Sundar Agrawal 37


RMO MATERIALS

218. (JMO2000) ABCD is a rectangle and line DX and DY and XY are drawn where X is on AB and Y
is on BC. The area of the triangle AXD is 5, the area of triangle BXY is 4 and area of CYD is 3.
Determine the area of triangle DXY.

219. (RMO2000) Let AC be a line segment in the plane and B a point between A and C. Construct
isosceles triangles PAB and QBC on one side of the segment AC such that
mAPB  mBQC  120 0 and an isosceles triangle RAC on the other side of AC such that
mARC  120 0 . Show that PQR is an equilateral triangle.
220. (RMO2000) The internal bisector of angle A in a triangle ABC with AC > AB, meets the circum
circle  of the triangle in D. Join D to the centre O of the circle  and suppose DO meets AC in E,
possibly when extended. Given that BE is perpendicular to AD, show that AO is parallel to BD.
221. (INMO2000) The incircle of a triangle ABC touches BC, CA, AB in K, L and M respectively. The
line through A and parallel to LK meets MK in P and the line through A and parallel to MK meets
Lm in Q. Show that the line PQ bisects the sides AB and AC of the triangle ABC.
222. (INMO2000) In a convex quadrilateral PQRS, PQ  RS , ( 3  1)QR  SP and
0 0
RSP  SPQ  30 , prove that PQR  QRS  90 .
   
223. (Canadian MO2000) Let ABCD be a quadrilateral with CBD  2 ADB, ABD  2CDB and
AB  CD , prove that AD  CD .
224. (IMO2000) AB is tangent to the circles CAMN and NMBD. M lies between C and D on the line
CD is parallel to AB. The chords NA and CM meet at P; the chords NB and MD meet at Q. The rays
CA and DB meet at E. Prove that PE = QE.
225. (IMO2000) A1 A2 A3 is an acute angled triangle. The foot of the altitude from Ai is K i and the
incircle touches the side opposite Ai at Li . The line K 1 K 2 is reflected in the line L1 L2 . Similarly
the line K 2 K 3 is reflected in the line L2 L3 and K 3 K 1 is reflected in the line L3 L1 . Show that the
three new lines form a triangle with vertices on the incircle.
226. (JMO2001) See the figure O is the centre of the circle. OBCD is a rectangle with OB = 19, BC = 90,
find AB = ?

227. (JMO2001) ABCD is a rectangle. The areas of 3 smaller enclosed rectangles are given in the figure.
Find out the area of shaded portion.

Dr. Shyam Sundar Agrawal 38


RMO MATERIALS

228. (JMO2001) A quadrilateral is circumscribed about a circle as shown in the figure. Find the value of
x.

229. (JMO2001) A right triangle ABC with hypotenuse AB has side AC = 15. Altitude CH divides AB
into segment AH and BH with HB = 16. Find the area of triangle ABC.

230. (RMO2001) Let BE and CF be the altitudes of an acute triangle ABC, with E on AC and F on AB.
Let O be the point of intersection of BE and CF. Take any line KL through O with K on AB and L
on AC. Suppose M and N are located on BE and CF respectively, such that KM is perpendicular to
BE and LN is perpendicular to CF. Prove that FM is parallel to EN.
231. (RMO2001) In a triangle ABC, D is a point on BC such that AD is the internal bisector of A .
Suppose B  2C and CD = AB. Prove that mA  72 0 .
232. (RMO2001) In the following figure a square is sub divided into 4 rectangles, out of which area of
two rectangles are given as 24 and 36. If sides of rectangle are integers, then find the area of the
square.

233. (RMO2001) In the figure AC is a diameter. BC is a tangent to the circle. If AC = 5, BD = 2AD, then
find the length of BC.

234. (RMO2001) In triangle ABC, AC = 12, CD = 9, AD = 6 and ABC  DAC . Find the perimeter of
triangle ABC.

Dr. Shyam Sundar Agrawal 39


RMO MATERIALS

235. (RMO2001) The length of sides of a triangle DEF are inversely proportional to the altitudes of
triangle ABC. Prove that lengths of altitude of triangle DEF are inversely proportional to length of
sides of triangle ABC.
236. (RMO2001) In triangle ABC, a  b  c and area of triangle is 1 sq unit. Prove that b  2 .
237. (INMO2001) Let ABC be a triangle and D be the midpoint of side BC. Suppose DAB  BCA
and DAC  150 . Show that ADC is obtuse. Further if O is the circumcentre of ADC, prove that
triangle AOD is equilateral.
238. (INMO2001) Let ABC be a triangle in which no angle is 90 0 . For any point P in the plane of the
triangle, let A1 , B1 , C1 denotes the reflection of P in the sides BC, CA and AB respectively. Prove the
following statements:
(a) If P is the incentre or an excentre of ABC, then P is the circumcentre of A1 B1C1 .
(b) If P is the circumcentre of ABC, then P is the orthocentre of A1 B1C1 .
(c) If P is the orthocentre or an excentre of ABC, then P is either the incentre or an excentre of
A1 B1C1 .
239. (IMO2001) Let ABC be an acute angled triangle with circum centre O. Let P on BC be the foot of
the altitude from A. Suppose that BCA  ABC  30 0 . Prove that CAB  COP  90 0 .
240. (IMO2001) In a triangle ABC, let AP bisect BAC , with P on BC, and let BQ bisect ABC , with
Q on CA. It is known that BAC  60 0 and that AB  BP  AQ  QB . What are the possible angles
of triangle ABC?
241. (JMO2002) In the given diagram what is the value of x?

242. (JMO2002) In the diagram adjacent edges are at right angles. The four longer edges are of equal
length and all of the shorter edges are also equal in length. The area of the shape is 528. What is the
perimeter?

243. (JMO2002) If the diameter of a circle is 5 units, find the area of an equilateral triangle inscribed in
the circle.
244. (JMO2002) What fraction of area of largest triangle to that of shaded one?

Dr. Shyam Sundar Agrawal 40


RMO MATERIALS

245. (JMO2002) In the diagram AD < BC. What is the perimeter of ABCD?

246. (RMO2002) In an acute angled triangle ABC, point D,E, F are located on sides BC, CA, AB
CD CA AE AB BF BC
respectively, such that  ,  ,  . Prove that AD, BE, CF are the
CE CB AF AC BD BA
altitudes of ABC.
247. (RMO2002) The circumference of a circle is divided into eight arcs by a convex quadrilateral
ABCD, with four arcs lying inside the quadrilateral and the remaining four lying outside it. The
lengths of the arcs lying inside the quadrilateral are denoted by p, q, r, s in counterclockwise
direction starting from some arc. Suppose p  r  q  s . Prove that ABCD is a cyclic quadrilateral.
248. (RMO2002) In a quadrilateral ABCD, E and F are two points on BC such that E is more nearer to B
than F. If mBAE  mCDF and mEAF  mFDE , then prove that mFAC  mEDB .
249. (RMO2002) In the following regular hexagon ABCDEF, two diagonals FC and BD intersect at G.
Find the ratio of area of quadrilateral DEFG to the area of triangle BGC.

250. (RMO2002) In an isosceles triangle ABC, AB = BC. From the circum centre of triangle ABC, a
perpendicular drawn to the CD (which is the angle bisector of C ), which intersects AB at F. A
line parallel to CD passes through E drawn, which intersect AB at F. Prove that BE = FD.
251. (RMO2002) In triangle ABC, O is the circum centre and G is the centroid. R and r be the radius of
in circle and circum circle of triangle ABC. Then prove OG  R ( R  2r ) .
252. (INMO2002) For a convex hexagon ABCDEF, consider the following six statements: (i) AB is
parallel to DE (ii) BC is parallel to EF (iii) CD is parallel to FA (iv) AE = BD (v) BF = CE (vi)
CA = DF.
(a) Show that if all the six statements are true, then the hexagon is cyclic.
(b) Prove that, in fact any five of these six statements also imply that hexagon is cyclic.
253. (IMO2002) BC is a diameter of a circle with centre O. A is any point on the circle with
AOC  60 0 . EF is the chord which is the perpendicular bisector of AO. D is the midpoint of the
minor arc AB. The line through O parallel to AD meets AC at J. Show that J is the incentre of the
triangle CEF.

Dr. Shyam Sundar Agrawal 41


RMO MATERIALS

254. (IMO2002) n  2 circles of radius 1 are drawn in the plane so that no line meets more than two of
1 
the circles. Their centres are O1 , O2 ,..., On . Show that   (n  1) .
i  j Oi O j 4
255. (JMO2003) ABCD and XYCZ each are square. If Area outside XYCZ and area inside ABCD is
30 cm 2 and DY  10cm , then find CD.

256. (JMO2003) In the given cuboids each surface area are 84 cm 2 , 70 cm 2 and 30 cm 2 . Find the volume
of cuboid.

257. (JMO2003) In the figure AB  AC , mBAD  30 0 , AE  AD . Find mCDE  ?

258. (JMO2003) Four congruent rectangles mixed and form a square. If perimeter of each rectangle is 16
then find the area of square ABCD.

259. (RMO2003) Let ABC be a triangle in which AB = AC and mCAB  90 0 . Suppose M and N are
points on the hypotenuse BC such that BM 2  CN 2  MN 2 . Prove that mMAN  45 0 .
260. Suppose P is an interior point of a triangle ABC such that the ratios
d ( A, BC ) d ( B, CA) d (C , AB)
  . Find the common value of these ratios. Here d(X, YZ) denotes
d ( P, BC ) d ( P, CA) d ( P, AB)
the perpendicular distance from X to line YZ.
261. (RMO2003) Lengths of sides of a triangle are integers. If perimeter of triangle is 8 then find its area.
262. (RMO2003) One right angled triangle is drawn by touching the circumference of a circle with radius
5. Find the length of its hypotenuse.

Dr. Shyam Sundar Agrawal 42


RMO MATERIALS

263. (RMO2003) From the figure find mA  mB  mC  mD  mE  mF  mG  ?

264. (RMO2003) Triangle ABC is an acute angled triangle with mA  30 0 . H is the point of
intersection of altitudes from the vertices A, B and C. Midpoint of BC is M. T lies on side HM such
that HM = MT. Prove that AT = 2BC.
BD 3
265. (RMO2003) From the given figure  and 6BE = AE, prove that 2AF = 9CF.
DC 4

266. (INMO2003) Consider an acute triangle ABC and let P be an interior point of ABC. Suppose the
lines BP and CP, when produced, meet AC and AB in E and F respectively. Let D be the point
where AP intersects the line segment EF and K be the foot of the perpendicular from D on to BC.
Show that DK bisects EFK .
267. (INMO2003) Let ABC be a triangle with sides a, b, c. Consider a triangle A1 B1C1 with sides equal to
b c a 9
a  , b  , c  . Show that [ A1 B1C1 ]  [ ABC ] , where [XYZ] means area of triangle XYZ.
2 2 2 4
268. (IMO2003) A convex hexagon has the property that for any pair of opposite sides the distance
between their midpoints is 3 / 2 times the sum of their lengths. Show that all the hexagon’s angles
are equal.
269. (IMO2003) ABCD is cyclic. The feet of the perpendicular from D to the lines AB, BC, CA are P,
Q, R respectively. Show that are angle bisector of ABC and CDA meet on the line AC iff RP = RQ.
270. (JMO2004) If measure of smallest angle of a triangle is 200 , then what is the at most measure of
largest angle of that triangle? (i) 80 0 (ii) 90 0 (iii) 140 0 (iv) 1590 0 (v) 160 0
271. (JMO2004) In the figure AE = 3, DE = 4, AD = 5. Then find the area of rectangle ABCD.

272. (JMO2004) 9 squares are like figure below such that it makes a rectangle whole area is 180 sq units.
Then find the perimeter of the rectangle.

Dr. Shyam Sundar Agrawal 43


RMO MATERIALS

273. (JMO2004) In the figure A and B are point lies on one side of a rectangle and joint by vertical point
of the rectangle such that length of two lines are 25 and 39. Small side of rectangle is 15 then find
length of AB.

274. (RMO2004) Consider in the plane a circle  with centre O and a line l not intersecting circle  .
Prove that there is a point Q on the perpendicular drawn from O to the line l , such that for any point
P on the line l , PQ represents the length of the tangent from P to the circle  .
275. (RMO2004) Let ABCD is a quadrilateral; X and Y be the mid points of AC and BD respectively and
the lines through X and Y respectively parallel to BD and AC meet in O. Let P, Q, R, S be the
midpoints of AB, BC, CD and DA respectively. Prove that:
(a) Quadrilateral APOS and APXS have the same area.
(b) The areas of quadrilateral APOS, BQOP, CROQ, and DSOR are all equal.
276. (RMO2004) Figure below contains 3 squares such that upper square is kept at the midpoints of side
of below two squares. If AB = 100 then find the area of each square.

277. (RMO2004) In the figure what part of Area of regular octagon is the area of triangle ABC?

278. (RMO2004) In triangle ABC, if BE  AC , CF  AB , K is the midpoint of EF and L is midpoint of


BC the prove that mKAE  mBAL .
279. (RMO2004) A, B, C are lies on a straight line. K is a point outside the straight line. If S1 , S 2 , S 3 are
the circum centre of triangle KAB, KAC and KBC respectively, then prove that K , S1 , S 2 , S 3 lies on
a circle.

Dr. Shyam Sundar Agrawal 44


RMO MATERIALS

280. (RMO2004) P is an interior point of an equilateral triangle ABC. If AP 2  BP 2  CP 2 , then prove


that mBPC  150 0 .
281. (INMO2004) Consider a convex quadrilateral ABCD in which K, L, M, N are the midpoints of the
sides AB, BC, CD, DA respectively. Suppose:
(a) BD bisects KM at Q.
(b) QA = QB = QC = QD
(c) LK/LM = CD/CB.
Prove that ABCD is a square.
282. (INMO2004) Let R denotes the circum radius of a triangle ABC; a, b, c its sides BC, CA, AB; and
ra , rb , rc its exradii opposite A, B, C. If 2 R  ra , prove that
(a) a  b and a  c .
(b) 2 R  rb and 2 R  rc .
283. (IMO2004) Let ABC be an acute angled triangle with AB  AC . The circle with diameter BC
intersects the sides AB and AC at M and N respectively. Denote O the midpoint of the side BC. The
bisectors of BAC and MON intersects at R. Prove that the circumcircles of the triangles BMR
and CNR have a common point lying on the side BC.
284. (IMO2004) Define a “hook” to be a figure made up of six unit squares as shown below in the
picture, or any of the figures obtained by applying rotation and reflections to this figure.

Determine all m  n rectangles that can be covered without gaps and without overlaps with hooks
such that :
(a) The rectangle is covered without gaps and without overlaps.
(b) No part of a hook covers area outside the rectangle.
285. (IMO2004) In a convex quadrilateral ABD the diagonal BD does not bisect the angles ABC and
CDA. The point P lies inside ABCD and satisfies PBC  DBA, PDC  BDA . Prove that
ABCD is a cyclic quadrilateral if and only if AP = CP.
286. (JMO2005) In the figure 3 congruent squares are there. If perimeter of the figure is 80 cm then find
its area?

287. (JMO2005) PQMN and ABCD two equal squares drawn like the figure such that centre of the
square PQMN is A and the side AB intersect 1/3 rd of side MN. Then shaded area is what portion of
the area of PQMN?

288. (JMO2005) In the figure mCAE  30 0 , mBAE  70 0 . Find mACD  ?

Dr. Shyam Sundar Agrawal 45


RMO MATERIALS

289. (RMO2005) Let ABCD be a convex quadrilateral; P, Q, R, S be the mid points of AB, BC, CD, DA
respectively such that AQR and CSP are equilateral. Prove that ABCD is a rhombus. Determine its
angles.
290. (RMO2005) In triangle ABC, let D be the mid point of BC. If mADB  450 and mACD  30 0 ,
determine mBAD .
291. (RMO2005) In the figure what part of Area of regular octagon is the area of rectangle ABCD.

292. (RMO2005) Perimeter of a triangle ABC is P and area is A and radius of its circum circle is R, then
P. A
find the maximum value of 3 .
R
293. (RMO2005) A Quadrilateral ABCD is circumscribed by a circle of radius 1 and AB.BC.CD.DA  4 .
Prove that ABCD is a square.
294. (RMO2005) M is an interior point of triangle ABC. Perimeter of triangle ABC is P. Prove that
P
 AM  MB  MC  P .
2
295. (INMO2005) Let M be the midpoint of side BC of a triangle ABC. Let the median AM intersect the
incircle of ABC at K and L, K being nearer to A than L. If AK = KL = LM. Prove that the sides of
triangle ABC are in the ratio 5 : 10 : 13 in some order.
296. (IMO2005) Six poit are chosen on sides of an equilateral triangle ABC. A1 , A2 on BC, B1 , B2 on
CA, and C1 , C 2 on AB, such that they are the vertices of a convex hexagon A1 A2 B1 B2 C1C 2 with
equal side lengths. Prove that the lines A1 B2 , B1C 2 and C1 A2 are concurrent.
297. (IMO2005) Let ABCD be a fixed convex quadrilateral with BC = DA and BC not parallel to DA.
Let two variable points E and F lie on the sides BC and DA, respectively and satisfy BE = DF. The
lines AC and BD meet at P, the lines BD and EF meets at Q, the lines EF and AC meet at R. Prove
that the circumcircles of the triangle PQR, as E and F vary, have a common point other than P.
298. (JMO2006) In the figure 3X3 square divided into 9, 1X1 squares. Find the area of the shaded
portion.

299. (JMO2006) In the figure four isosceles right triangles arranged to form a square. If area of white
portion is 18 sq units, then find the area of shaded portion (sum of area of four isosceles right
triangles).

Dr. Shyam Sundar Agrawal 46


RMO MATERIALS

300. (JMO2006) In the figure mABC  mBDC  mDEC  90 0 . If mBCA  27 0 , then find
mBDE  ?

301. (JMO2006) ABCD is a rectangle. P and Q are centers of the circle. Radius of each circle is 6 cm,
RS = 2 cm. Find area of rectangle ABCD.

302. (RMO2006) Let ABC be an acute angled triangle and let D, E, F be the foot of perpendiculars from
A, B, C respectively to BC, CA, AB. Let the perpendiculars from F to CB, CA, AD, BE meets them
in P, Q, M, N respectively. Prove that P, Q, M, N are collinear.
303. (RMO2006) Let ABCD be a quadrilateral in which AB is parallel to CD and perpendicular to AD;
AB = 3CD and the area of the quadrilateral is 4. If a circle can be drawn touching all the sides of the
quadrilateral. Find its radius.
304. (RMO2006) In triangle ABD, AB = BC, Altitude AD intersect circum circle at P. Prove that
AP.BC  2 AB.BP .
305. (RMO2006) In triangle ABC, D and E are two points on Ac and AB respectively. BD and CE
AD 1 BE 1
intersect at X, such that  and  . Prove that area of triangle AXC is two-third of area
DC 2 AE 3
of triangle ABC.
1 1
306. (RMO2006) In the given figure how to draw BPC through P such that value of  is
BP PC
maximum.

307. (INMO2006) In a non equilateral triangle ABC, the sides a, b, c forms an arithmetic progression. Let
I and O denote the incentre and circumcentre of the triangle respectively.
Dr. Shyam Sundar Agrawal 47
RMO MATERIALS

(a) Prove that IO is perpendicular to BI.


(b) Suppose BI extended meets AC in K, and D, E are the midpoints of BC, BA respectively. Prove
that I is the circumcentre of triangle DKE.
308. (INMO2006) In a cyclic quadrilateral ABCD, AB = a, BC = b, CD = c,
ABC  120 0 , ABD  30 0 . Prove that
(a) c  a  b
(b) c  a  c  b  c  a  b
309. (IMO2006) Let ABC be a triangle with incentre I. A point P in the interior of the triangle satisfies
PBA  PCA  PBC  PCB . Show that AP  AI , and that equality holds if and only if
PI.
310. (JMO2007) Find area of the convex quadrilateral ABCD, given AB = 15, BC = 20, CD = 7, AD =
24 and AC = 25.

311. (JMO2007) The points A, B, C and D are placed consecutively on a circle with diameter AC so that
AB = 7 cm, BC = 24 cm and CD = 15 cm. What is the area of the quadrilateral ABCD?

312. (JMO2007) In the figure CD is a diameter of the semicircle with centre at O, AB = OD and
mDOE  450 , then compute mBOE .

313. (JMO2007) In the diagram at right segments join the vertices of a unit square to the midpoints of its
sides. Find the area of the shaded quadrilateral?

314. (RMO2007) Let BAC be an acute angled triangle; Ad is the bisector of BAC with D on BC, and
BE be the altitude from B to AC. Show that mCED  45 0 .
315. (RMO2007) A trapezium ABCD, in which AB is parallel to CD, is inscribed in a circle with centre
O. Suppose the diagonals AC and BD of the trapezium intersects at M and OM = 2.
Dr. Shyam Sundar Agrawal 48
RMO MATERIALS

(i) If mAMB  60 0 , find with proof, the difference between the lengths of the parallel sides.
(ii) If mAMD  60 0 , find with proof, the difference between the lengths of the parallel sides.
316. (INMO2007) In a triangle ABC right angled at C, the median through B bisects the angle between
5 AB
BA and the bisector of B . Prove that:  3 .
2 BC
317. (INMO2007) Let ABC be a triangle in which AB = AC. Let D be the midpoint of BC and P be a
AP BP BD
point on AD. Suppose E is the foot of the perpendicular from P on AC. If   , m
PD PE AD
and z  m 2 (1   ) , prove that z 2  (3  2  2) z  1  0 . Hence show that   2 and   2 if only
if ABC is equilateral.
318. (IMO2007) Consider five points A, B, C, D, E such that ABCD is a parallelogram and BCED is a
cyclic quadrilateral. Let  be a line passing through A. Suppose that  intersects the interior of the
segment DC at F and intersects line BC at G. Suppose also that EF  EG  EC . Prove that  is the
bisector of angle DAB.
319. (IMO2007) In triangle ABC the bisector of angle BCA intersects the circumcircle again at R, the
perpendicular bisector of BC at P, and the perpendicular bisector of AC at Q. The midpoint of BC is
K and the midpoint of AC is L. Prove that the triangle RPK and RQL have the same area.
320. (JMO2008) A 3X8 rectangle is cut into two pieces along the line AB as shown in the figure. The
two pieces are then rearranged to form a right angled triangle. What is the perimeter of the triangle
formed?

321. (JMO2008) In the figure below AB and CD are parallel lines. Find value of x.

322. (JMO2008) In the figure below what is the sum of angles marked 1, 2, 3, 4, 5, 6, 7 and 8.

323. (RMO2008) Let BAC be an acute angled triangle; let D, F be midpoints of BC and AB respectively.
Let the perpendicular from F to AC and the perpendicular from B to BCmeet in N. Prove that ND is
equal to the circum radius of ABC.
Dr. Shyam Sundar Agrawal 49
RMO MATERIALS

324. (INMO2008) Let ABC be a triangle, I is its incentre; A1 , B1 , C1 be the reflections of I in BC, CA, AB
respectively. Suppose the circum circle of triangle A1 B1C1 passes through A. Prove that B1 , C1 , I , I1
are concyclic, where I1 is the incentre of triangle A1 B1C1 .
325. (INMO2008) Let ABC be a triangle A , B , C be three equal disjoint circles inside ABC such that
A touches AB, B touches BC and CA. Let  be a circle touching circles A , B , C externally.
Prove that the line joining the circumcentre O and the incentre I of triangle ABC passes through the
centre of  .
326. (IMO2008) An acute angled triangle ABC has orthocenter H. The circle passing through H with
centre the midpoint of BC intersects the line BC at A1 and A2 . Similarly, the circle passing through
H with centre the midpoint of CA intersects the line CA at B1 and B2 , and the circle passing
through H with centre the midpoint of AB intersects the line AB at C1 and C 2 show that
A1 , A2 , B1 , B2 , C1 , C 2 lies on a circle.
327. (IMO2008) Let ABCD be a convex quadrilateral with BA  BC . Denote the incircles of triangles
ABC and ADC by w1 and w2 respectively. Suppose that there exist a circle w tangent to the ray
BA beyond A and to the ray BC beyond C, which is also tangent to the lines AD and CD. Prove that
the common external tangents w1 and w2 intersect on w .
328. (JMO2009) ABC is a triangle with the length of sides as integer in the ratio 3:4:5.
(a) If one of the side of ABC is 28, then what are other sides?
(b) Find the length of the sides of ABC which has perimeter 96.
329. (RMO2009) Let ABC be a triangle in which AB = AC and let I be its in-centre. Suppose BC = AB
+ AI. Find BAC .
330. (RMO2009) A convex polygon  is such that the distance between any two vertices of  does not
exceed 1.
(a) Prove that the distance between any two points on boundary of  does not exceed 1.
(b) If X and Y are two distinct points inside  , prove that there exist a point Z on the boundary of
 such that XZ  YZ  1 .
331. (INMO2009) Let ABC be a triangle and let P be an interior point such that BPC  90 0 ,
BAP  BCP . Let M, N be the midpoints of AC and BC respectively. Suppose BP = 2PM. Prove
that A, P, N are collinear.
332. (INMO2009) Let ABC be an acute angled triangle and let H be its orthocentre. Let hmax denote the
largest altitude of the triangle ABC. Prove that AH  BH  CH  2hmax .
333. (IMO2009) Let ABC be a triangle with circumcentre O. The points P and Q are interior points of
the sides CA and AB, respectively. Let K, L and M be the midpoints of the segments BP, CQ and
PQ respectively, and let  be the circle passing through K, L and M. Suppose the line PQ is tangent
to the circle  . Prove that OP = OQ.
334. (IMO2009) Let ABC be a triangle with AB = AC. The angle bisector of CAB and ABC meet
the sides BC and CA at D and E respectively. Let K be the incentre of triangle ADC. Suppose that
BEK  45 0 ,. Find all possible values of CAB .
335. (JMO2010) In the figure below determine the sum of angles: A, B, C , D, E , F .

Dr. Shyam Sundar Agrawal 50


RMO MATERIALS

336. (JMO2010) In the figure the two triangles drawn are equilateral triangles. What is the value of x?

337. (JMO2010) In triangle ABC, AC = BC, the bisector of angle BAC meets BC at E, and AE is
perpendicular to BC. Determine the measures of all angles of triangle.
338. (JMO2010) In the adjoining figure the regular hexagon has been divided into four trapezia and one
hexagon. If each of five sections has the same perimeter, what is the ratio of the lengths p, q and r?

339. (RMO2010) Let ABCDEF be a convex hexagon in which the diagonals AD, BE, CF are concurrent
at O. Suppose the area of triangle OAF is the geometric mean of those of OAD and OEF, and the
area of triangle OBC is the geometric mean of those of OAB and OCD. Prove that the area of
triangle OED is the geometric mean of those of OCD and OEF.
340. (RMO2010) Let ABC be a triangle in which mA  60 0 . Let BE and CF are bisectors of B and
C with E on AC and F on AB. Let M be the reflection of A in the line EF. Prove that M lies on
BC.
341. (INMO2010) Let ABC be a triangle with circum circle  . Let M be a point in the interior of triangle
ABC which is also on the bisector of A . Let AM, BM, CM meet  in A1 , B1 , C1 respectively.
Suppose P is the point of intersection of A1C1 with AB; and Q is the point of intersection of
A1 B1 with AC. Prove that PQ is parallel to BC.
342. (INMO2010) Let ABC be an acute angled triangle with altitude AK. Let H be its orthocentre and O
be its circumcentre. Suppose KOH is an acute angled triangle and P its circumcentre. Let Q be the
reflection of P in the line HO. Show that Q lies on the line joining the mid point of AB and AC.
343. (IMO2010) Let I be the incentre of triangle ABC and let  be its circumcircle. Let the line AI
intersect  again at D. Let E be a point on the arc BDC and F a point on the side BC such that

Dr. Shyam Sundar Agrawal 51


RMO MATERIALS

1
BAF  CAE  BAC . Finally, let G be the mid point of the segment IF. Prove that the lines
2
DG and EI intersect on  .
344. (IMO2010) Let P be a point inside the triangle ABC. The lines AP, BP and CP intersect the
circumcircle  of triangle ABC again at the points K, L and M respectively. The tangent to  at C
intersect the line AB at S. Suppose that SC = SP. Prove that MK = ML.
345. (JMO2011) In the diagram XYZ is isosceles with XY = XZ. Also, point W is on XZ so that
XW  WY  YZ . Determine the measure of XYZ , XYW .

346. (JMO2011) Squares ABCD and EFGH are equal in area. Vertices B, E, C and H on the same line.
Diagonal AC is extended to join, the midpoint of GH. Determine ratio of the shaded portion of the
two squares.

347. (JMO2011) In the trapezium ABCD, AC = BD. Show that AD = BC.

348. (JMO2011) In the diagram, PR, PS, QS, QT and RT are straight line segments. QT intersects PR and
PS at U and V respectively. If PU = PV, UPV  24 0 , PSQ  x 0 , TQS  y 0 , find the value of
x y .

349. (RMO2011) Let ABC be a triangle. Let D, E, F be points respectively on the segments BC, CA, AB
BD BF
such that AD, BE, CF concur at point K. Suppose  and ADB  AFC . Prove that
DC FA
ABE  CAD .
Dr. Shyam Sundar Agrawal 52
RMO MATERIALS

350. (RMO2011) Let ABC be a triangle and let BB1 , CC1 be respectively the bisector of B, C with
B1 on AC and C1 on AB. Let E, F be the feet of perpendiculars drawn from A onto BB1 and
CC1 respectively. Suppose D is the point at which the incircle of ABC touches AB. Prove that
AD  EF .
351. (INMO2011) Let D, E, F be points on the sides of BC, CA, AB respectively of a triangle ABC such
that BD  CE  AF and BDF  CED  AFE . Prove that ABC is equilateral.
352. (INMO2011) Let ABCD be a quadrilateral inscribed in a circle  . Let E, F, G, H be the mid points
of the arcs AB, BC, CD, and DA of the circle  . Suppose AC.BD  EG.FH . Prove that AC, BD,
EG, FH are concurrent.
353. (IMO2011) Let ABC be an acute angled triangle with circumcircle  . Let  be a tangent to  , and
let  a ,  b ,  c be the lines obtained by reflecting  in the lines BC, CA and AB respectively. Show
that the circumcircle of the triangle determined by the lines  a ,  b ,  c is tangent to the circle  .
354. (JMO2012) In ABC , AD  BC . BC = 8 cm and AD = 6 cm. If E and F are mid points of BD and
AC respectively find EF.

355. (JMO2012) Find the area of the shaded square.

356. (JMO2012) In rectangle ABCD we have AD = 1, P is on AB, and DB and DP trisect ADC. What is
the perimeter of DBDP?
357. (PreRMO2012) A triangle with perimeter 7 has integer side lengths. What is the maximum possible
area of such a triangle?
358. (PreRMO2012) In a triangle ABC, we have AC = BC = 7 and AB = 2, Suppose that D is a point on
the line AB such that B lies between A and D and CD = 8. What is the length of the segment BD?
359. (PreRMO2012) In rectangle ABCD, AB = 5 and BC = 3. Points F and G are on line segment CD so
that DF = 1 and GC = 2. Lines AF and BG intersect at E. What is the area of triangle AEB?
360. (PreRMO2012) ABCD is a square and AB = 1. Equilateral triangles AYB and CXD are drawn such
that X and Y are inside the square. What is the length of XY?
361. (PreRMO2012) O and I are the circumcentre and incentre of triangle ABC respectively. Suppose O
lies in the interior of triangle ABC and I lies on the circle passing through B, O, C. What is the
magnitude of BAC in degrees?
362. (PreRMO2012) PS is a line segment of length 4 and O is the midpoint of PS. A semicircular arc is
drawn with PS as diameter. Let X be the midpoint of this arc. Q and R are points on the arc PXS
such that QR is parallel to PS and the semicircular arc drawn with QR as diameter is tangent to PS.
What is the area of the region QXROQ bounded by the two semicircular arcs?

Dr. Shyam Sundar Agrawal 53


RMO MATERIALS

363. (RMO2012) Let ABC be a triangle. Let D be a point on segment BC such that DC = 2BD. Let E be
BP AP
the mid point of AC. Let AD and BE intersect in P. Determine the ratio and .
PE PD
364. (RMO2012) Let ABC be a triangle and let BE, CF be respectively internal angle bisector of
B, C with E on AC and F on AB. Let X is a point on the segment CF such that AX  CF and
Y is a point on the segment BE such that AY  BE . Prove that XY  (b  c  a) / 2 , where
a  BC , b  CA, c  AB .
365. (RMO2012) Let ABCD be a unit square. Draw a quadrant of a circle with A as centre and B, Das
end points of the arc. Similarly draw a quadrant of a circle with B as centre and A, C as end points
of the arc. Inscribe a circle  touching the arc AC internally, the arc BD internally and also
touching the side AB. Find the radius of the circle  .
366. (RMO2012) Let ABC be a triangle. Let D, E be a points on the segment BC such that BD = DE =
EC. Let F be the mid point of AC. Let BF intersect AD in P and AE in Q respectively. Determine
BP/PQ.
367. (RMO2012) Let ABC be a triangle. Let D, E be a points on the segment BC such that BD = DE =
EC. Let F be the mid point of AC. Let BF intersect AD in P and AE in Q respectively. Determine
the ratio of the area of the triangle APQ to that of the quadrilateral PDEQ.
368. (INMO2012) Let ABCD be a quadrilateral inscribed in a circle. Suppose AB  2  2 and AB
subtend 1350 at the centre of the circle. Find the maximum possible area of ABCD.
369. (INMO2012) Let ABC be an acute angled triangle, let D, E, F be points on BC, CA, AB
respectively such that AD is the median, BE is the internal angle bisector and CF is the altitude.
Suppose FDE  C , DEF  A and EFD  B . Prove that ABC is equilateral.
370. (IMO2012) Given triangle ABC the point J is the centre of the excircle opposite the vertex A. This
excircle is tangent to the side BC at M, and to the lines AB and AC at K and L respectively. The
lines LM and BJ meet at F, and the lines KM and CJ meet at G. Let S be the point of intersection of
the lines AF and BC, and let T be the point of intersection of lines AG and BC. Prove that M is the
midpoint of ST.
371. (IMO2012) Let ABC be a triangle with BCA  90 0 , and let D be the foot of the perpendicular
from C. Let X be a point in the interior of the segment CD. Let K be the point on the segment AX
such that BK = BC. Similarly, let L be the point on the segment BX such that AL = AC. Let M be
the point of intersection of AL and BK. Show that MK = ML.
372. (JMO2013) From the figure A  B  C  D  E  F  G  H  I  J  ?

373. (JMO2013) In the figure, CD and BD are the bisectors of the interior angle C and exterior angle
B of triangle ABC. They meet each other at D. DM is parallel to BC and meets AB at L. If LB = 5
cm and CM = 7 cm, Find length of LM.

Dr. Shyam Sundar Agrawal 54


RMO MATERIALS

374. (JMO2013) What fraction of the diagram is shaded?

375. (JMO2013) The top left corner of the given square is joined to the mid point of the bottom edge, the
mid point of the top edge is joined to the bottom right corner; the top right corner is joined to the
midpoints of the left edge, and the mid point of the right edge is joined to the bottom left corner.
What fraction of square is shaded?

376. (JMO2013) In ABC , AD bisects BAC and meets BC at D and CE bisects ACB and meets AD
at E. If AB = 4 cm, AC = 5 cm, CD = 3 cm and area of the triangle ABC is 18cm2. Find the area of
the triangle AEC.

377. (PreRMO2013) Three points X, Y, Z are on a straight line such that XY = 10 and XZ = 3. What is
the product of all possible values of YZ?
378. (PreRMO2013) Let AD and BC be the parallel sides of a trapezium ABCD. Let P and Q be the
midpoints of the diagonals AC and BD. If AD = 16 and BC = 20, what is the length of PQ?
379. (PreRMO2013) In a triangle ABC, let H, I and O be the orthocenter, incenre and circumcentre
respectively. If the points B, H, I, C lie on a circle, what is the magnitude of BOC in degrees?
380. (PreRMO2013) Let ABC be an equilateral triangle. Let P and S be points on AB and AC,
respectively, and let Q and R be points on BC such that PQRS is a rectangle. If PQ  3PS and the
area of PQRS is 28 3 , what is the length of PC?
381. (PreRMO2013) Let A1 , B1 , C1 , D1 be the midpoints of the sides of a convex quadrilateral ABCD and
let A2 , B2 , C 2 , D 2 be the midpoints of the sides of the quadrilateral A1 B1C1 D1 . If A2 B2 C 2 D2 is a
rectangle with sides 4 and 6, then what is the product of the lengths of the diagonal of ABCD?
382. (PreRMO2013) Let S be a circle with centre O. A chord AB, not a diameter, divides S into two
regions R1 , R2 such that O belongs to R2 . Let S 1 be a circle with centre R1 , touching AB at X and S
internally. Let S 2 be a circle with centre R2 , touching AB at Y, the circle S internally and passing
through the centre S. The point X lies on the diameter passing through the centre of S 2 and
YXO  30 0 . If the radius of S 2 is 100 then what is the radius of S 1 ?

Dr. Shyam Sundar Agrawal 55


RMO MATERIALS

383. (PreRMO2013) In a triangle ABC with BCA  90 0 , the perpendicular bisector of AB intersects
segments AB and AC at X and Y, respectively. If the ratio of the area of the quadrilateral BXYC to
the area of the triangle ABC is 13:18 and BC = 12 then what is the length of AC?
384. (RMO2013) Let ABC be an isosceles triangle with AB = AC and let  denotes its circumcircle. A
point D is on arc AB of  not containing C. A point E is on arc AC of  not containing B. If AD =
CE prove that BE parallel to AD.
385. (RMO2013) In a triangle ABC, points D and E are on segments BC and AC such that BD = 3DC
and AE = 4EC. Point P is on line ED such that D is the midpoint of the segment EP. Lines AP and
BC intersect at point S. Find the ratio BS/SD.
386. (RMO2013) Let ABC be an acute-angled triangle. The circle  with BC as diameter intersects AB
and AC again at P and Q respectively. Determine BAC given that the orthocenter of triangle APQ
lies on  .
387. (RMO2013) Let ABC be a triangle with A  90 0 and AB = AC. Let D and E be points on the
segment BC such that BD : DE : EC  3 : 4 : 5 . Prove that DAE  45 0 .
388. (RMO2013) In a triangle ABC, AD is the altitude from A and H is the orthocenter. Let K be the
centre of the circle passing through D and tangent to BH at H. Prove that the line DK bisects AC.
389. (RMO2013) Let ABC be a triangle with A  90 0 and AB = AC. Let D and E be points on the
segment BC such that BD : DE : EC  1 : 2 : 3 . Prove that DAE  45 0 .
390. (RMO2013) In an acute angled triangle ABC with AB < AC, the circle  touches AB at B and
passes through C interesting AC again D. Prove that the orthocenter of triangle ABD lies on  if
and only if and only if it lies on the perpendicular bisector of BC.
391. (RMO2013) Let ABC be a triangle which is not right angled. Define a sequence of triangles
Ai Bi C i , with i  0 , as follows: A0 B0 C 0 is the triangle ABC; and for i  0 , Ai 1 , Bi 1 , C i 1 are the
reflections of the orthocenter of triangle Ai Bi C i in the sides Ai Bi , C i Ai , Ai Bi respectively. Assume
that Am  An for some distinct natural numbers m, n. Prove that A  60 0 .
392. (RMO2013) Let  be a circle with centre O. Let  be another circle passing through O and
intersecting  at points A and B. A diameter CD of  intersects  at a point P different from O.
Prove that APC  BPD .
393. (RMO2013) In a triangle ABC, let H denote its orthocenter. Let P be the reflection of A with
respect to BC. The circumcircle of triangle ABP intersects the line BH again at Q, and the
circumcircle of triangle ACP intersects the line CH again at R. Prove that H is the incentre of
triangle PQR.
394. (INMO2013) Let 1 and 2 be two circles touching each other externally at R. Let l1 be a line
which is tangent to 2 at P and passing through the centre O1 of 1 . Similarly let l 2 be a line which
is tangent to 1 at Q and passing through the centre O2 of 2 . Suppose l1 and l 2 are not parallel and
intersect at K. If KP  KQ , prove that the triangle PQR is equilateral.
395. (INMO2013) In an acute triangle ABC, O is the circum centre, H is the orthocenter and G is the
centroid. Let OD be perpendicular to BC and HE be the perpendicular to CA, with D on BC and E
on CA. Let F be the mid point of AB, suppose the areas of triangles ODC, HEA and GFB are equal.
Find all possible values of C .
396. (IMO2013) Let the ex-circle of triangle ABC opposite the vertex A be tangent to the side BC at the
point A1 . Define the points B1 on CA and C1 on AB analogously, using the ex-circles opposite B
and C, respectively. Suppose that the circumcentre of the triangle A1 B1C1 lies on the circumcircle of
the triangle ABC. Prove that triangle ABC is right angled.

Dr. Shyam Sundar Agrawal 56


RMO MATERIALS

397. (JMO2014) Four identical circles are enclosed in a square of area 16 cm2 as shown in the figure.
Find out the area of the shaded portion.

398. (JMO2014) In the diagram AQ, BR and CP are drawn from the vertices A, B and C of triangle ABC.
Points P, Q and R lie on AQ, BR, and CP respectively. If AP = 3PD, BQ = 3QR, CR = 3PR, then
determine the ratio between the areas of triangle PQR and triangle ABC.

399. (JMO2014) In the given below diagram, ABCD is a square and APB is a right angle. If
CQ || AP, DQ || BP and X is a point on CQ, such that P lies on BX, then prove that the measure of
PQX  450 .

400. (PreRMO2014) Let ABCD be a convex quadrilateral with perpendicular diagonals. If Ab = 20, BC
= 70, and CD = 90, then what is the value of DA?
401. (PreRMO2014) In a triangle with integer side lengths, one side is three times as long as a second
side, and the length of the third side is 17. What is the greatest possible perimeter of the triangle?
402. (PreRMO2014) In a triangle ABC, X and Y are points on the segments AB and C, respectively, such
that AX:XB = 1:2 and AY : YC = 2:1. If the area of triangle AXY is 10 then what is the area of
triangle ABC?
403. (PreRMO2014) Let ABCD be a convex quadrilateral with DAB  BDC  90 0 . Let the incircles
of triangles ABD and BCD touch BD at P and Q respectively, with P lying in between B and Q. If
AD = 999 and PQ = 200 then what is the sum of the radii of the incircles of triangles ABD and
BDC?
404. (PreRMO2014) Let XOY be a triangle with XOY  90 0 , let M and N be the midpoints of legs OX
and OY, respectively. Suppose that XN = 19 and YM = 22. What is XY?
405. (PreRMO2014) In a triangle ABC, I denote the incentre, Let the lines AI, BI, Ci intersects the
incircle at P, Q and R respectively. If BAC  40 0 , what is the value of QPR in degrees?
406. (RMO2014) Let ABC be an acute angled triangle in which ABC is the largest angle. Let O be its
circumcentre. The perpendicular bisectors of BC and AB meet AC at X and Y respectively. The
internal bisector of AXB and BYC meet AB and BC at D and E respectively. Prove that BO is
perpendicular to AC if DE is parallel to AC.

Dr. Shyam Sundar Agrawal 57


RMO MATERIALS

407. (RMO2014) Let D, E, F be the points of contact of the incircle of an acute angled triangle ABC
with BC, CA, AB respectively. Let I 1 , I 2 , I 3 be the incentres of the triangle AFE, BDF, CED,
respectively. Prove that the lines I 1 D, I 2 E , I 3 F are concurrent.
408. (RMO2014) Let ABC be a triangle and let AD be the perpendicular from A on to BC. Let K, L, M
be points on AD such that AK = AL = LM = MD. If the sum of the areas of the shaded regions is
equal to the sum of areas of the unshaded regions, prove that BD = DC.

409. (RMO2014) Let ABC be an acute angled triangle and let H be its ortho-centre. For any point P on
the circum-circle of triangle ABC, let Q be the point of intersection of the line BH with the line AP.
Show that there is a unique point X on the circum-circle of ABC such that for every point P  A, B ,
the circum-circle of HQP pass through X.
410. (RMO2014) In an acute angled triangle ABC, ABC is the largest angle. The perpendicular
bisectors of BC and BA intersect AC at X and Y respectively. Prove that circumcentre of triangle
ABC is incentre of triangle BXY.
411. (RMO2014) Let ABC be a triangle with AB  AC . Let P be a point on the line AB beyond A such
that AP  PC  AB . Let M be the point mid-point of BC and let Q be the point on the side AB such
that CQ  AM . Prove that BQ  2 AP .
412. (RMO2014) Let ABC be an acute-angled triangle and ABC is the largest angle of the triangle.
Let R be its circumcentre. Suppose the circumcircle of triangle ARB cuts AC again in X. Prove that
RX is perpendicular to BC.
413. (RMO2014) Let ABC be a triangle. Let X be on the segment BC such that AB  AX . Let AX meet
the circum circle  of triangle ABC again at D. Show that the circum centre of BDX lies on  .
414. (RMO2014) Let ABCD be an isosceles trapezium having an incircle; let AB and CD be the parallel
sides and let CE be the perpendicular from C on to AB. Prove that CE is equal to the geometric
mean of AB and CD.
415. (RMO2014) Let ABC be an acute angled triangle and let I be the incentre. Let the incircle of
triangle ABC touch BC in D. The incircle of the triangle ABD touches AB in E; the incircle of the
triangle ACD touches BC in F. Prove that B, E, I, F are concyclic.
416. (RMO2014) What is the minimal area of a right angled triangle whose inradius is 1 unit?
417. (INMO2014) In a triangle ABC, let D be a point on the segment BC such that AB + BD = AC +
CD. Suppose that the points B, C and the centroids of triangles ABD and ACD lie on a circle. Prove
that AB = AC.
418. (INMO2014) In an acute angled triangle ABC, a point D lies on the segment BC. Let O1 ,O2 denote
the circumcentres of the triangles ABD and ACD respectively. Prove that the line joining the
circumcentre of triangle ABC and the orthocenter of triangle O1O2 D is parallel to BC.
419. (IMO2014) Convex quadrilateral ABCD has ABC  CDA  90 0 . Point H is the foot of the
perpendicular from A to BD. Points S and T lie on sides AB and AD, respectively, such that H lies
inside triangle SCT and CHS  CSB  90 0 , THC  DTC  90 0 . Prove that line BD is tangent
to the circumcircle of triangle TSH.
420. (IMO2014) A point P and Q lie on side BC of acute-angled triangle ABC so that PAB  BCA
and CAQ  ABC . Points M and N lie on lines AP and AQ respectively, such that P is the

Dr. Shyam Sundar Agrawal 58


RMO MATERIALS

midpoint of AM and Q is the midpoint of AN. Prove that lines BM and CN intersect on the
circumcircle of triangle ABC.
421. (JMO2015) In the diagram below, ABC and PQR each is an equilateral triangle. If BPQ  30 0 and
each side of triangle ABC is of length 12 cm, determine the area of triangle PQR.

422. (JMO2015) In the diagram below, ABCD is a square, and triangle PBC is an equilateral triangle. If
BD is the diagonal of the square and each side of the square is 8 3 cm long, determine the area of
triangle PBD correct to one place of decimal (take 3  1.732 ).

423. (JMO2015) The rectangle shown has length AC = 32 units, width AE = 20 units, and B and F are
midpoints of AC and AE respectively. Find the area of the quadrilateral ABDF.

424. (JMO2015) Five straight lines intersect at a common points and five triangles are constructed as
shown. What is total of the 10 angles marked on the diagram?

425. (JMO2015) Area of ABCD is 25.9 cm2. Find the area of the ring.

426. (JMO2015) ABCD is a square of area 1 3 sq units. BEC is an equilateral triangle. AC meets BE
at F, find the area of triangle BFC.

Dr. Shyam Sundar Agrawal 59


RMO MATERIALS

427. (PreRMO2015) The figure below shows a broken piece of a circular plate made of glass. C is the
midpoint of AB, and D is the midpoint of arc AB. Given that AB = 24 cm and CD = 6 cm, what is
the radius of the plate in centimetres? (The figure is not drawn to scale.)

428. (PreRMO2015) What is the greatest possible perimeter of a right-angled triangle with integer side
lengths if one of the sides has length 12?
429. (PreRMO2015) In rectangle ABCD, AB = 8 and BC = 20. Let P be a point on AD such that
BPC  90 0 . If r1 , r2 , r3 are the radii of the incircles of triangles APB, BPC and CPD, what is the
value of r1  r2  r3 ?
430. (PreRMO2015) In acute-angled triangle ABC, let D be the foot of the altitude from A, and E be the
midpoint of BC. Let F be the midpoint of AC. Suppose BAE  40 0 . If DAE  DFE , what is
the magnitude of ADF in degrees?
431. (PreRMO2015) The circle ω touches the circle Ω internally at P. The centre O of Ω is outside ω. Let
XY be a diameter of Ω which is also tangent to ω. Assume PY > PX. Let PY intersect ω at Z. If YZ
= 2PZ, what is the magnitude of PYX in degrees?
432. (RMO2015) Let ABCD be a convex quadrilateral with AB  a, BC  b, CD  c, DA  d . Suppose
a 2  b 2  c 2  d 2  ab  bc  cd  da , and the area of ABCD is 60 square units. If the length of
one of the diagonals is 30 units, determine the length of the other diagonal.
433. (RMO2015) Let ABC be a right angled triangle with B  90 0 and let BD be the altitude from B
on AC. Draw DE  AB and DF  BC . Let P, Q, R and S be respectively the incentres of triangle
DFC, DBF, DEB and DAE. Suppose S, R, Q are collinear. Prove that P, Q, R, D lie on a circle.
434. (RMO2015) In a cyclic quadrilateral ABCD, let the diagonals AC and BD intersect at X. Let the
circumcircles of triangles AXD and BXC intersect again at Y. If X is the incentre of triangle ABY,
show that CAD  90 0 .
435. (RMO2015) Let ABC be a right triangle with B  90 0 . Let E and F be respectively the mid points
of AB and AC. Suppose the incentre I of triangle ABC lies on the circumcircle of triangle AEF. Find
the ratio BC/AB.
436. (RMO2015) Let ABC be a triangle. Let B  and C  denote respectively the reflection of B and C in
the internal angle bisector of A . Show that the triangle ABC and AB C  have the same incentre.
437. (RMO2015) Two circles  and  in the plane intersect at two distinct points A and B, and the
centre of  lies on  . Let points C and D be on  and  , respectively, such that C, B and D are
collinear. Let point E on  be such that DE is parallel to AC. Show that AE = AB.
438. (RMO2015) Two circles  and  with centres O and O  , respectively, are such that O  lies on
 . Let A be a point on  and M mid-point of the segment AO  . If B is the point on  different
from A such that AB is parallel to OM, show that the midpoint of AB lies on  .
439. (RMO2015) Let ABC be a triangle with circum circle  and incentre I. Let the internal angle
bisector of A , B and C meet  in A , B  , C  respectively. Let B C  intersect AA in P and
AC in Q, and let BB  intersect AC in R. Suppose the quadrilateral PIRQ is a kite; that is IP = IR and
QP = QR. Prove that ABC is an equilateral triangle.
440. (RMO2015) Let ABC be a triangle. Let B  denote the reflection of B in the internal angle bisector
 of A . Show that the circumcentre of the triangle CB I lies on the line  , where I is the incentre
of ABC.
Dr. Shyam Sundar Agrawal 60
RMO MATERIALS

441. (RMO2015) Two circles  and  in the plane intersect at two distinct points A and B. A line
through B intersects  and  again in C and D, respectively. Suppose CA = CD. Show that the
centre of  lies on  .
442. (INMO2015) Let ABC be a right-angled triangle with B  90 0 . Let BD be the altitude from B on
to AC. Let P, Q and I be the incentres of triangles ABD, CBD and ABC respectively. Show that the
circumcentre of the triangle PIQ lies on the hypotenuse AC.
443. (INMO2015) Let ABCD be a convex quadrilateral. Let the diagonals AC and BD intersect in P. Let
PE, PF, PG and PH be the altitudes from P on to the sides AB, BC, CD and DA respectively. Show
1 1 1 1
that ABCD has an incircle if and only if    .
PE PG PF PH
444. (IMO2015) Let ABC be an acute triangle with AB  AC . Let  be its circum circle, H its
orthocenter, and F the foot of the altitude from A. Let M be the midpoint of BC. Let Q be the point
on  such that HQA  90 0 , and let K be the point on  such that HKQ  90 0 . Assume that the
points A, B, C, K and Q are all different, and lie on  in this order. Prove that the circumcircles of
triangles KQH and FKM are tangent to each other.
445. (IMO2015) Triangle ABC has circumcircle  and circumcentre O. A circle  with centre A
intersects the segment BC at points D and E, such that B, D, E and C are all different and lie on line
BC in this order. Let F and G be the points of intersection of  and  , such that A, F, B, C and G
lie on  in this order. Let K be the second point of intersection of the circumcircle of triangle BDF
and the segment AB. Let L be the second point of the intersection of the circumcircle of triangle
CGE and the segment CA. Suppose that the lines FK and GL are different and intersect at the point
X. Prove that X lies on the line AO.
446. (JMO2016) Three congruent circles each of radius 14 cm with centre A, B, C respectively are drawn
such that each passes through the centre of the other two circles as shown in the figure. Find the area
of the shaded portion.

447. (JMO2016) In the figure triangle PQR has PQ = PR, QPS  30 0 . If PS = PT, then determine
mTSR .

448. (JMO2016) In the given figure, OXY is the ¼ th of a circle representing one quadrant. Taking OX
and OY as diameters semicircles are drawn. The area of shaded portions are written in the from A
and B. The find the ratio of A and B.

Dr. Shyam Sundar Agrawal 61


RMO MATERIALS

449. (JMO2016) ABCD is a square. E and F are points on AD and CD respectively. The area of triangle
ABE is 5 square units. The area of triangle BCF is 4 square units. The area of triangle DEF is 3
square units. Find the area of the triangle BEF.

450. (JMO2016) In the given diagram parallelogram ABCD has been divide into four parallelograms PQ
and SR drawn parallel to BC and AB respectively and O is the point of intersection of PQ and SR.
Prove that the area of SOQD – area of BPOR = 2 X area of triangle AOC.

451. (JMO2016) In the given figure, the angle marked as a, b, c, d, e, f are congruent. Find the measure
of each of them.

452. (JMO2016) In triangle ABC, C is a right angle and CD is perpendicular to AB. Prove that:
1 1 1
2
 2
 .
CD AC BC 2
453. (JMO2016) In triangle ABC, ABC  ACB  780 , D and E are the points on AB and AC
respectively such that BCD  240 and CBE  510 , find BED .
454. (PreRMO2016) Let AD be an altitude in a right triangle ABC with A  90 0 and D on BC. Suppose
that the radii of the incircles of the triangles ABD and ACD are 33 and 56 respectively. Let r be the
radius of the incircle of triangle ABC. Find the value of 3(r + 7)
455. (PreRMO2016) In triangle ABC right angled at vertex B, a point O is chosen on the side BC such
that the circle γ centered at O of radius OB touches the side AC. Let AB = 63 and BC = 16, and the
radius of γ be of the form m n where m, n are relatively prime positive integers. Find the value of m
+ n.
456. (PreRMO2016) The hexagon OLYMPI has a reflex angle at O and convex at every other vertex.
Suppose that LP = 3√2 units and the condition∠O = 10∠L = 2∠Y = 5∠M = 2∠P = 10∠I holds.
Find the area (in sq units) of the hexagon.
457. (PreRMO2016) Points G and O denote the centroid and the circumcenter of the triangle ABC.
Suppose that AGO  90 0 and AB = 17, AC = 19. Find the value of BC 2 .
458. (RMO2016) Let ABC be scalene, with BC as the largest side. D be the foot of the altitude from A
to side BC. Let points K and L be chosen on the lines AB and AC respectively, such that D is the
midpoint of the segment KL. Prove that the points B, K, C, L are concyclic if and only if
mBAC  90 0 .

Dr. Shyam Sundar Agrawal 62


RMO MATERIALS

459. (RMO2016) ABC is an equilateral triangle with side length 11 units. As shown in the fugure, points
P1 , P2 ,..., P10 are taken on sides BC I that order; dividing the side into 11 segments on unit length
each. Similarly points Q1 , Q 2 ,..., Q10 are taken on sides CA and points R1 , R 2 ,..., R10 are taken on
side AB. Count the number of triangles of the form Pi Q j R k such that their centroid coincides with
the centroid of ABC . (Each of the indices i, j , k is chosen from {1, 2, …, 10}, and need not be
distinct.)

460. (RMO2016) Let ABC be a triangle and D be the mid point of BC. Suppose the angle bisector of
ADC is tangent to the circumcircle of triangle ABD at D. Prove that A  90 0 .
461. (RMO2016) Let ABC be a right angled triangle with B  90 0 . Let I be the incentre of ABC.
Extend AI and CI; let them intersect BC in D and AB in E respectively. Draw a line perpendicular to
AI at I to meet AC in J; draw a line perpendicular to CI at I to meet AC in K. Suppose DJ = EK.
Prove that BA = BC.
462. (RMO2016) Let ABC be a right angled triangle with B  90 0 , let I be the incentre of ABC. Draw
a line perpendicular to AI at I. Let it intersects the line CB at D. Prove that CI is perpendicular to
AD and prove that ID  b(b  a ) where BC = a, and CA = b.
463. (RMO2016) Let ABC be a triangle with centroid G. Let the circumcircle of triangle AGB intersect
the line BC in X different from B; and the circumcircle of triangle AGC intersect the line BC in Y
different from C. Prove that G is the centroid of the triangle AXY.
464. (RMO2016) Given are two circle w1 , w2 which intersects at points X, Y. Let P be an arbitrary point
on w1 . Suppose that the lines PX, PY meet w 2 again at points A, B respectively. Prove that the
circumcircles of all triangles PAB have the same radius.
465. (RMO2016) Two circles C1 , C 2 intersects each other at points A and B. Their external common
tangent (closer to B) touches C1 at P and at Q. Let C be the reflection of B in line PQ. Prove that
CAP  BAQ .
466. (INMO2016) Let ABC be a triangle in which AB  AC . Suppose the orthocenter of the triangle lies
on the incircle. Find the ratio AB / BC .
467. (INMO2016) Let ABC be a right angled triangle with B  90 0 . Let D be a point on AC such that
the inradii of the triangle ABD and CBD are equal. If this common value is r  and r is the inradii
1 1 1
of triangle ABC, prove that   .
r  r BD
468. (IMO2016) Triangle BCF has a right angle at B. Let A be the point on line CF such that FA = FB
and F lies between A and C. Point D is chosen such that DA = DC and AC is the bisector of DAB .
Point E is chosen such that EA = ED and AD is the bisector of EAC . Let M be the midpoint of
CF. Let X be the point such that AMXE is a parallelogram (where AM || EX and AE || MX ). Prove
that lines BD, FX and ME are concurrent.
469. (JMO2017) In the given figure find the sum of the vertex angles.
Dr. Shyam Sundar Agrawal 63
RMO MATERIALS

470. (JMO2017) In the given figure ABCDEFGH is an octagon comprising of 16 congruent rectangles of
size 4 X 1 each. P is a point on EF such that AP divides the octagon into two parts of equal area.
Find the length of AP.

471. (JMO2017) In the square ABCD, E and F are the points on the opposite side of the diagonal BD,
BEF  EFD  90 0 , BE = 14, EF = 3 and FD = 7 units, find the area of the square.

472. (JMO2017) In the given figure ABC is a right triangle with B  90 0 , BC = 8 cm, AB = 4 cm.
BDEF is a square inscribed in it. Find the side of the square.

473. (JMO2017) In the given figure a circle of radius 28 cm is divided into 8 equal parts. Taking the
same centre another circle of radius 14 cm has been drawn. Find the area of the shaded portion.

474. (PreRMO2017) In a rectangle ABCD, E is the midpoint of AB; F is a point on AC such that BF is
perpendicular to AC; and FE perpendicular to BD. Suppose BC  8 3 . Find AB.
475. (PreRMO2017) Let P be an interior point of a triangle ABC whose side lengths are 26, 65, 78. The
line through P parallel to BC meets AB in K and AC in L. The line through P parallel to CA meets
BC in M and BA in N. The line through P parallel to AB meets CA in S and CB in T. If KL, MN,
ST are of equal lengths, find this common length.
476. (PreRMO2017) Let ABCD be a rectangle and let E and F be points on CD and BC respectively such
that area(ADE) = 16, area(CEF)=9 and area(ABF) = 25. What is the area of triangle AEF?
477. (PreRMO2017) Let AB and CD be two parallel chords in a circle with radius 5 such that the centre
O lies between these chords. Suppose AB =6 , CD =8. Suppose further that the area of the part of the
Dr. Shyam Sundar Agrawal 64
RMO MATERIALS

circle lying between the chords AB and CD is (m  n) / k , where m, n, k are positive integers with
gcd(m, n, k) = 1. What is the value of m + n + k?
478. (PreRMO2017) Let  1 be a circle with centre O and let AB be a diameter of  1 . Let P be a point
on the segment OB different from O. Suppose another circle  2 with centre P lies in the interior of
 1 . Tangents are drawn from A and B to the circle  2 intersecting  1 again at A1 and B1
respectively such that A1 and B1 are on the opposite sides of AB. Given that A1 B  5, AB  15 and
OP = 10, find the radius of  1 .
479. (PreRMO2017) Consider the areas of the four triangles obtained by drawing the diagonals AC and
BD of a trapezium ABCD. The product of these areas, taken two at time, are computed. If among
the six products so obtained, two products are 1296 and 576, determine the square root of the
maximum possible area of the trapezium to the nearest integer.
480. (RMO2017) Let AOB be given angle less than 1800 and let P be an interior point of the angular
region determined by AOB . Show, with proof, how to construct, using only ruler and compasses,
a line segment CD passing through P such that C lies on the ray OA and D lies on the ray OB, and
CP:PD = 1:2.
481. (RMO2017) Let  be a circle with a chord AB which is not a diameter. Let 1 be a circle on one
side of AB such that it is tangent to AB at C and internally tangent to  at D. Likewise, let 2 be a
circle on other side of AB such that it is tangent to AB at E and internally tangent to  at F.
Suppose the line DC intersects  at X  D and the line FE intersects  at Y  F . Prove that XY
is a diameter of  .
482. (INMO2017) In the given figur, ABCD is a square sheet of paper. It is folded along EF such that A
goes to a point A different from B and C, on the side BC and D goes to D  . The line AD  cuts CD
in G. Show that the inradius of the triangle GCA is the sum of inradii of the triangle GD F and
ABE .

483. (INMO2017) Let ABC be a triangle with A  90 0 and AB  AC . Let AD be the altitude from A
onto BC. Let P, Q and I denote respectively the incentres of triangles ABD, ACD and ABC. Prove
that AI is perpendicular to PQ and AI = PQ.
484. (INMO2017) Let ABCDE be a convex pentagon in which A  B  C  D  120 0 and side
lengths are five consecutive integers in some order. Find all possible values of AB+BC+CD.
485. (IMO2017) Let R and S be different point on a circle  such that RS is not a diameter. Let l be the
tangent line to  at R. Point T is such that S is the midpoint of the line segment RT. Point J is
chosen on the shorter arc RS of  so that the circumcircle  of triangle JST intersects l at two
distinct points. Let A be the common point of  and l that is closer to R. Line AJ meets  again at
K. Prove that the line KT is a tangent to  .
486. (PreRMO2018) In a quadrilateral ABCD, it is given that AB = AD = 13, BC = CD = 20,BD= 24. If r
is the radius of the circle inscribable in the quadrilateral, then what is the integer closest to r?
487. (PreRMO2018) Let ABCD be a trapezium in which AB || CD and AD  AB . Suppose ABCD has
an incircle which touches AB at Q and CD at P. Given that PC= 36 and QB = 49, find PQ.

Dr. Shyam Sundar Agrawal 65


RMO MATERIALS

488. (PreRMO2018) A point P in the interior of a regular hexagon is at distances 8816 units from three
consecutive vertices of the hexagon, respectively. If r is radius of the circumscribed circle of the
hexagon, what is the integer closest to r?
489. (PreRMO2018) Let AB be a chord of a circle with centre O. Let C be a point on the circle such that
ABC  30 0 and O lies inside the triangle ABC. Let D be a point on AB such that
DCO  OCB  20 0 . Find the measure of CDO in degrees.
490. (PreRMO2018) In a triangle ABC, the median from B to CA is perpendicular to the median from C
to AB. If the median from A to BC is 30, determine ( BC 2  CA 2  AB 2 ) / 100 .
491. (PreRMO2018) In a triangle ABC, right-angled at A, the altitude through A and the internal bisector
of A have lengths 3 and 4, respectively. Find the length of the median through A.
492. (PreRMO2018) Triangles ABC and DEF are such that A  D , AB = DE = 17, BC = EF = 10 and
AC - DF = 12. What is AC + DF?
493. (PreRMO2018) Let ABC be an acute-angled triangle and let H be its orthocentre. Let G1 , G 2 , G 3 be
the centroids of the triangles HBC, HCA and HAB, respectively. If the area of triangle G1G 2 G3 is 7
units, what is the area of triangle ABC?
494. (PreRMO2018) Let D be an interior point of the side BC of a triangle ABC. Let I 1 , I 2 be the
incentres of triangles ABD and ACD respectively. Let AI1 , AI 2 meet BC in E and F respectively. If
BI 1 E  60 0 , what is the measure of CI 2 F in degrees?
495. (RMO2018) LET ABC be a triangle with integer sides in which AB<AC. Let the tangent to the
circumcircle of triangle ABC at A intersect the line BC at D. Suppose AD is also an integer. Prove
that gcd(AB, AC)>1.
496. (RMO2018) Let ABC be an acute angled triangle with AB<AC. Let I be the incentre of triangle
ABC, and let D, E, F be the points st which its touches the sides BC, CA, AB respectively. Let BI,
CI meets the line EF at Y, X respectively. Further assume that both X and Y are outside the triangle
ABC. Prove that:
(a) B, C, Y, X are concyclic;
(b) I is also the incentre of triangle DYX.
497. (INMO2018) Let ABC be non-equilateral triangle with integer sides. Let D and E be respectively
the mid-points of BC and CA; let G be the centroid of triangle ABC. Suppose D, C, E, G are
concyclic. Find the least possible perimeter of triangle of ABC.
498. (INMO2018) Let 1 and 2 be two circles with respective centres O1 and O 2 intersecting in two
distinct points A and B such that O1 AO2 is an obtuse angle. Let the circumcircle of triangle
O1 AO2 intersects 1 and 2 respectively in points C and D. Let the line CB intersects 2 in E; let
the DB intersects 1 in F. Prove that the points C, D, E, F are concyclic.
499. (IMO2018) Let be the cicumcircle  of the acute angled triangle ABC. Points D and E lie on the
segments AB and AC respectively, such that AD = AE. The perpendicular bisectors of SD and CE
intersect the minor arc AB and AC of  at point F and G respectively. Prove that the lines DE and
FG are parallel (or on the same line).
500. (IMO2018) A convex quadrilateral ABCD satisfies AB.CD = BC.DA. Point X lies inside ABCD so
that XAB  XCD and XBC  XDA . Prove that BXA  DXC  1800 .

Dr. Shyam Sundar Agrawal 66


RMO MATERIALS

Dr. Shyam Sundar Agrawal 67

Das könnte Ihnen auch gefallen